You are on page 1of 120

Made by: Shahd A.

Gaber

(Test 6-1)
Question: 1
Correct Answer: F
Explanation:
In the description of the Coevolution Hypothesis, the passage states that RNA
sequences developed enzyme-like abilities including the ability to self-replicate.
Development of capsids was mentioned in the Regressive Evolution Hypothesis, but not
in Coevolution, eliminating choice (G). DNA was not mentioned in the Coevolution
Hypothesis, eliminating choice (H). Cell membrane transit was mentioned only in the
Cellular Origin Hypothesis, eliminating choice (J).

Question: 2
Correct Answer: D
Explanation:
The first paragraph states that biologists agree that viruses originated from nucleic acid.
Choice (A) is eliminated because the Coevolution Hypothesis does not provide an
explanation for viruses evolving from bacteria. Choice (B) is eliminated because
according to the passage, viruses cannot replicate unless they first infect a cell. Choice
(C) is eliminated because only the Regressive Evolution Hypothesis and Coevolution
Hypothesis mentions anything about viruses having membranes or envelopes.
Question: 3
Correct Answer: J
Explanation:
The Coevolution Hypothesis specifically mentions that the first virus particles were RNA
nucleotides with enzyme-like activity that incorporated into protein structures. Therefore,
choices (F), (G), and (H) are all eliminated. DNA is not mentioned as a component of
the earliest virus particles in this hypothesis.

Question: 4
Correct Answer: B
Explanation:
The Cellular Origin Hypothesis states that viruses originated from cellular-organism
ancestors. They could have come from prokaryotic or eukaryotic organisms, and the
type of ancestor cell does not necessarily determine what type of cell they will evolve to
infect. Therefore, choices (C) and (D) are eliminated because they cannot necessarily
be determined from the information given. Given that both T4 and PP7 infect bacteria,
they likely contain genetic material similar to the bacteria they infect. However, given
that one is a DNA virus and one is an RNA virus, if they are related at all it is likely
Made by: Shahd A.Gaber

through a very distant ancestor virus that escaped a cellular organism long ago. This
eliminates choice (A) and makes choice (B) the best answer.

Question: 5
Correct Answer: H
Explanation:
Since the Regressive Evolution Hypothesis suggests that viruses originated from
obligate intracellular parasites, one would expect that they would share features of
these ancestors. The hypothesis specifically states that intracellular parasites evolved
into viruses, eliminating choices (F) and (G). Between the remaining choices, finding a
parasite with DNA similar to a known virus could suggest that this organism and the
known virus evolved from a common ancestor, making choice (H) the best answer.

Question: 6
Correct Answer: B
Explanation:
The passage states that viruses cannot replicate unless they first infect a cell.
Therefore, the first viruses were unlikely to originate before cells, making choice (B) the
best answer. There is nothing in the passage to support that viral capsids are similar to
bacterial cell walls, and the word "capsid" does not even appear in the first hypothesis,
eliminating choice (A). Choice (C) is eliminated because the differences in RNA and
DNA viruses are not elaborated in the passage. Moreover, given that RNA viruses are
mentioned in the Coevolution Hypothesis and DNA viruses are mentioned in the
hypotheses that relate to more complex cellular organisms, it seems unlikely that RNA
viruses would be considered more advanced than DNA viruses. Choice (D) is
eliminated because the Coevolution Hypothesis specifically states that the first viruses
contained RNA.

Question: 7
Correct Answer: F
Explanation:
The incorporation of RNA sequences into protein structures versus cell structures is
specifically mentioned in the Coevolution Hypothesis. Choices (G) and (J) relate to the
Regressive Evolution Hypothesis and choice (H) relates to the Cellular Origin
Hypothesis.
Made by: Shahd A.Gaber

(Test 7-2)
Question: 6
Correct Answer: J
Explanation:
Looking at Figure 1, since 800 mg of sulfamethoxazole results in greater bacterial
elimination than 400 mg over time, choices (F) and (G) are both eliminated. Table 1
provides insight to the mechanism of action of sulfamethoxazole, so choice (H) is also
eliminated. According to Figure 1, combining sulfamethoxazole (SMX) with either
doxycycline or azithromycin increases overall elimination of a common skin infection
bacterium, making choice (J) the best answer.

Question: 7
Correct Answer: A
Explanation:
According to Figure 1, it takes 120 minutes for an 800 mg dose of sulfamethoxazole to
achieve 80% bacterial elimination and hence 20% original bacterial survival. At the
same time, the 400 mg dose only achieves between 40-50% elimination. A 600 mg
dose would be expected to give results between these two values at the 120 minute
mark. Therefore, only choice (A) is possible because the time required to reach 20%
survival would be greater than 120 minutes for a 600 mg dose.

Question: 8
Correct Answer: F
Explanation:
According to the passage and the data in Figure 1, penicillin is not very effective against
the bacterium studied, but it does eliminate some bacteria, eliminating choices (H) and
(J). In the experiment in the passage, less than 10% of the bacteria are eliminated by
penicillin after 120 minutes (2 hours), meaning most survived. Therefore, choice (F) is
better than choice (G).

Question: 9
Correct Answer: D
Explanation:
Figure 1 shows greater bacterial elimination by doxycycline and azithromycin when
compounded with sulfamethoxazole (SMX). This eliminates choices (A) and (B). Choice
(C) is a true statement. However, even though sulfamethoxazole 800 mg is more
effective than SMX/azithromycin according to the figure, this does not answer the
Made by: Shahd A.Gaber

question of whether or not azithromycin is more effective when combined


with sulfamethoxazole. Therefore, choice (D) is the best answer.

Question: 10
Correct Answer: H
Explanation:
The passage describes the goal of antibiotics as eliminating bacteria, so choices (F)
and (G) are incorrect. From the data in Figure 1, it can be inferred that antibiotics that
eliminate a large amount of bacteria in a short amount of time are most effective. This
eliminates choice (J) and makes choice (H) the best answer.

(Test 8-2)
Question: 7
Correct Answer: B
Explanation:
B Since all four blood types were produced in equal measure in Analysis 3, refer back
to Table 1 to determine your ratio. There are two blood types that contain at least one
IA allele: A and AB. Similarly, there are two blood types that contain at least one
IB allele: B and AB. Since there are two possible blood types for each allele, the ratio of
possibilities must be 1:1.

Question: 8
Correct Answer: H
Explanation:
H If you’re not sure what codominance is, look to the description of the
experiment: When an individual has 1 IA and 1 IB allele, this individual will have type-AB
blood, due to the codominance of the IA and IB alleles. Consequently, an offspring
whose genotype exhibits codominance must have type-AB blood. Look to Table 1 to
see that the genotype for type-AB blood is IAIB, choice (H).

Question: 9
Correct Answer: C
Explanation:
C Table 1 indicates that the only genotype that can produce type AB blood is I AIB. For
this to happen in every offspring, and because each parent contributes only a single
allele, one parent must only be capable of contributing IA while the other parent is only
capable of contributing IB. The cross of IAIA × IBIB will always produce offspring with
IAIB genotypes and type AB blood phenotypes. If you chose choice (A), be careful—
Made by: Shahd A.Gaber

think of all possible combinations of these two genotypes. If one parent contributes
IB and the other parent contributes IB, the offspring will have type-B blood.

Question: 10
Correct Answer: G
Explanation:
G Since both parents come from Analysis 1, refer back to the description there: One
thousand males with type-O blood were mated with 1,000 females with type-AB
blood. In Analysis 1, the males had only Genotype IOIO and the females had only
Genotype IAIB. The only possible combinations of these genotypes are IAIO and IBIO.
Since it is these offspring who are mated in Analysis 3, their blood types must be
IAIO and IBIO

Question: 11
Correct Answer: D
Explanation:
D If you’re not sure how to find the answer, use process of elimination. The easiest
place to look for blood-types containing IO alleles would be in analyses that produced
offspring with type-O blood. This includes Analyses 2 and 3, so any answer choice that
does not contain both can be eliminated—eliminate choices (A) and (C). Since one of
the parents in Analysis 1 has type-O blood (Genotype IOIO), one of these alleles must
have been passed on to each of the offspring—eliminate choice (B). This leaves only
choice (D), so even if you’re not sure about Analysis 4, choice (D) is the only answer
that contains all the others about which you are sure. Note: Analysis 4 works because
the parents had genotypes IAIO and IBIB, so their offspring could have only genotypes
IBIO and IAIB.

Question: 12
Correct Answer: H
Explanation:
H Analysis 3 had offspring with 4 possible phenotypes in equal frequency. If 300
offspring were identified, then 25% of 300 or 75 of them would be expected to have type
B blood.
Made by: Shahd A.Gaber

(Test 9-1)
Question: 1
Correct Answer: B
Explanation:
B Table 3 shows that a pumped blood volume of 450 mL yields a % BP of 8.8, while a
pumped blood volume of 500 mL yields a % BP of 9.3. The % BP of 9.0 is between
these other two % BP values, so it was likely produced by a pumped blood volume
between 450 and 500 mL. Only choice (B) fits this requirement.

Question: 2
Correct Answer: F
Explanation:
F Table 2 shows that when the artificial heartbeat pumps blood at 60 beats per minute,
the % BP is 1.2. As the rate at which the blood is pumped increases, so does the %
BP, so it follows that as the rate decreases, so would the % BP. Therefore, choice (F)
is the best answer.

Question: 3
Correct Answer: A
Explanation:
A Experiment 1 shows that as the artificial heart beat rate increases, so does the %
BP, and only choices (A) or (B) show this. The passage states that a lower blood
pressure follows from faster blood flow through the site of vasoconstriction. Since %
BP is calculated by subtracting the pressure at the site of vasoconstriction from normal
blood pressure, % BP increases with a lower pressure at the site of vasoconstriction.
Thus, an increase in velocity causes an increase in % BP. Choice (A) summarizes this.

Question: 4
Correct Answer: J
Explanation:
J According to the passage, a faster velocity of blood flow and a lower blood pressure
are consistent with a narrow region of a blood vessel, while a slower velocity of blood
flow and a higher blood pressure are consistent with a wide region of a blood vessel.
Therefore, the measurement at location A would have been taken at the region with the
widest diameter, while the measurement at location B would have been taken at the
region with the narrowest diameter. Choice (J) best illustrates this. Note: Since the
values in the chart are not listed in a consistent order, you can safely eliminate choices
(F) and (G), which show the three diameters consistently increasing from A–C or
decreasing from A–C, respectively.
Made by: Shahd A.Gaber

Question: 5
Correct Answer: C
Explanation:
C Experiments 2 and 3 use an artificial heart that pumps blood at a constant rate of 90
beats per minute. In Experiment 3, the % BP is measured as 9.3 for a blood volume of
500 mL, the amount that was used in Experiment 1 for a heart rate of 90 beats per
minute. In Experiment 2, a % BP of 9.3 was measured when the diameter of the site of
vasoconstriction was 0.8 cm. Therefore, it follows that the diameter of the site of
vasoconstriction used throughout Experiment 3 was also 0.8 cm, so choice (C) is the
best answer.

Question: 6
Correct Answer: G
Explanation:
G As the diameter of the site of vasoconstriction increases, the % BP decreases. Refer
back to the passage for the % BP formula. From this formula, you can deduce that a
larger % BP will result from a larger difference between normal pressure and pressure
at the site of vasoconstriction. Only the graph in choice (G) shows this pattern across
the three given diameters. If you selected choice (F) be careful—you may have chosen
them in the opposite order.

(Test 13-1)
Question: 1
Correct Answer: B
Explanation:
B Note this from the passage: If an enzyme cannot be produced, then the product of
the reaction that the enzyme catalyzes cannot be synthesized and the reactant in that
reaction will become highly concentrated. In other words, if a particular chemical cannot
produce the appropriate enzyme, that chemical then becomes highly concentrated.
Look at Figure 1 to see which enzyme catalyses the reaction of which isocitrate is the
reactant: Enzyme 2. Therefore, since yeast that cannot produce Enzyme 2 will have the
highest concentration of isocitrate, choice (B) is the correct answer.

Question: 2
Correct Answer: H
Explanation:
H The passage tells you that a gene notated with a superscript positive sign is not
damaged and that a gene notated with a superscript negative sign is damaged. The
passage tells you that if one of the four genes in Table 2 is damaged, then the enzyme
it is responsible for cannot be produced. Therefore Cat1+ Cat2- Cat3- Cat4+ means that
Made by: Shahd A.Gaber

Enzyme 1 and Enzyme 4 can be produced and Enzyme 2 and Enzyme 3 cannot be
produced, so the correct answer is choice (H).

Question: 3
Correct Answer: B
Explanation:
B The passage tells you that in the first reaction, citrate is the reactant and isocitrate is
the product. Based on this, you know that the reactant is what the arrow points away
from and the product is what the arrow points to. Now, take a look at the relationship
shown in Figure 1 between the three chemicals in the question (citrate, isocitrate, and α-
ketoglutarate). You can see that α-ketoglutarate is a product of a reaction involving
isocitrate, and isocitrate is a product of a reaction involving citrate, choice (B).

Question: 4
Correct Answer: G
Explanation:
G Based on Table 1, you know yeast strain X grows in growth media where α-
ketoglutarate, succinyl-CoA, and succinate are added, but cannot grow when citrate or
isocitrate are added. This means that the succinate synthesis reaction pathway can
happen only if α-ketoglutarate or any reactant after that in the pathway is added, so
there is something which prevents the reaction pathway from proceeding from isocitrate
to α-ketoglutarate. Therefore, choice (G) is the correct answer.

Question: 5
Correct Answer: A
Explanation:
A A control is anything in a given experiment that is left alone. Note in Table 1 that BNS
appears in all instances, once by itself and then with a number of chemicals added to it.
It can be reasonably inferred, then, that the basic nutrition solution (BNS) alone is the
control.

Question: 6
Correct Answer: J
Explanation:
J To answer this question, figure out which strains of yeast have a Yes in the row
where succinyl-CoA is the added chemical: strains W, X, and Y. Now, all you need to do
is look at which other basic nutrition solution + additional chemical all three strains grew
in. The only other medium that all three grow in as well is BNS + succinate, so choice
(J) is the answer.
Made by: Shahd A.Gaber

(Test 13-2)
Question: 7
Correct Answer: B
Explanation:
B Examine Figure 1: the incidence of virus A cycles annually with the greatest
incidence around June and July of each year. Therefore, the incidence of virus A is
greatest during the summer season.

Question: 8
Correct Answer: F
Explanation:
F From Figure 1, the rate of incidence is least where the number of cases is least for
the population. During April 2001, virus B and virus D exhibit the greatest number of
cases. Therefore, answer choices (G) and (J) may be eliminated. Between viruses A
and C, virus C exhibits the greater amount, which eliminates answer choice (H).

Question: 9
Correct Answer: A
Explanation:
A Examine the data for viruses B and C closely in Figure 1. Virus B, unlike the other
viruses, gradually increases in prevalence over the time of the survey. Virus C,
however, cycles seasonally. During the first two months of the survey, the incidence of
virus C exceeds that of virus B. Following February 2000, the incidence of virus B is
consistently greater than that of virus C. Consequently, only answer choice (A) provides
data that contradicts the virologist’s statement.

Question: 10
Correct Answer: G
Explanation:
G Examine each of the months given as answers. During April 2000 and November
2001, the incidences of virus B and virus D are similar; however the incidences of virus
A and C differ considerably. In contrast, during September 2000, three of the lines
appear to meet at about 5 cases per 1,000 individuals surveyed with only one incidence
value differing considerably. Answer choice (J) is similar to choices (F) and (H), in that
only two lines appear to intersect. Therefore, answer choice (G) exhibits the most
similar incidences of viral infection.
Made by: Shahd A.Gaber

Question: 11
Correct Answer: C
Explanation:
C Look at Figure 2: The number of deaths attributed to virus A is approximately 7 per
1,000 individuals during May and August. Looking at the number of deaths attributed to
virus D during May and September, the number is approximately 2 per 1,000
individuals, which is consistent with the values given in the context of the question. Of
the answer choices, only May is present as a choice. Therefore, the answer should be
choice (C).

(Test 15-1)
Question: 1
Correct Answer: B
Explanation:
B Choice (B) is correct because it is the only choice that Figure 1 says is necessary for
the mosquito life cycle to progress. If there is no water then the eggs will not hatch.

Question: 2
Correct Answer: G
Explanation:
G To solve this problem, you need to average the five numbers in the Percent of Group
Affected by Yellow Fever column. 10% + 18 % + 29% + 38% + 52% = 147%. 147% ÷ 5
= 29.4%, which is closest to 30%.

Question: 3
Correct Answer: A
Explanation:
A This question asks you to look at trends in Table 1 and Figure 2 to determine
amounts of mosquito bites per month. Figure 2 tells you about trends in cases of Yellow
Fever. Table 1 tells you that as the number of mosquito bites increases, cases of Yellow
Fever increase. Now look at Figure 2: if you know that more mosquito bites means more
cases of Yellow Fever, then the month with the most mosquito bites is most likely to be
the month with the most cases of Yellow Fever. April has 6 cases, June has 2 cases,
August and November each have 1 case, which means that choice (A), April, is the
correct answer.
Made by: Shahd A.Gaber

Question: 4
Correct Answer: G
Explanation:
G Choice (G) is the correct answer because February and March had the same amount
of rainfall. To solve this problem you must compare the amounts of rainfall for each of
the 2 months in the answer choices. Only one answer, choice (G), has the same
amount for the 2 months in the pair. This question tests your ability to read a graph with
2 y-axes; be sure to check which axis you’re getting your information from. For this
question you should use the one on the right, rainfall (inches.)

Question: 5
Correct Answer: D
Explanation:
D To solve this problem, you must determine the trend of the numbers in the number of
monkeys Seen column. This question is tricky because the numbers increase some and
decrease some but don’t do either consistently. If the answer is not entirely right, it must
be wrong, so you can eliminate choice (A) because the numbers don’t only increase,
choice (B) because the numbers don’t only decrease, and choice (C) because the
numbers did not increase then decrease, they partially decreased, then increased, then
decreased again, then increased again. It’s intimidating to choose choice (D), varied
with no consistency, but it is the only correct answer.

(Test 16-1)
Question: 1
Correct Answer: F
Explanation:
F Student 1 hypothesizes that vegetative reproduction and seed distribution together
make up the only ways that Taraxicum reproduces. The experiment described would
prevent vegetative reproduction (glass jars) and seed distribution (plastic bags), so if
Student 1 is correct, no daughter plants should ever grow.

Question: 2
Correct Answer: A
Explanation:
A Trials 2 and 3 both describe instances in which the experimenters have interfered
with one of the ways that dandelions spread themselves. The first trial, in
which Taraxicum specimens are left to grow naturally, provides the control for this
experiment, meant to establish what happens without any change in the variables being
investigated.
Made by: Shahd A.Gaber

Question: 3
Correct Answer: G
Explanation:
G Choice (G) is correct, while choice (F) states the opposite of what Student 2 claims—
the actual claim was that dandelions have a single taproot, unlike vegetative
reproducers. Choice (H) is wrong: the plant holds the seeds loosely not because they
are not important, but because they must be distributed for the plant to spread. Choice
(J) is wrong because no claim is made that vegetative reproducers never produce
seeds.

Question: 4
Correct Answer: B
Explanation:
B Student 2 says that Taraxicum does use vegetative reproduction … to replace the
above-ground plant if it has been cut or lost, and that this allows Taraxicum to survive
threats in the environment. Choice (A) is Student 1’s argument, which Student 2 rejects.
Choice (C) is rejected by Student 1 and never mentioned by Student 2. Choice (D) is an
observation made by Student 1, but Student 2 does not comment on whether it is
accurate.

Question: 5
Correct Answer: F
Explanation:
F Student 1 and Student 2 believe that dandelions spread through both seed
distribution and vegetative reproduction; they disagree only on the relative importance
of the two means. Thus, choice (F) is correct because neither means of reproduction is
limited. Choices (G) and (H) both describe situations in which some part of the
dandelion’s reproductive ability is experimentally tampered with, while choice (J)
describes a situation in which both the dandelion’s reproductive strategies were
negated.

Question: 6
Correct Answer: C
Explanation:
C Student 1 asserts that vegetative reproduction and seed distribution each make up
equal parts of Taraxicum’s reproduction. Experiment 3 is meant to remove vegetative
reproduction from the picture, while leaving seed distribution. If choice (D) is true, then
student 2 is right; if choice (A) is true, then dandelions do not rely on seed distribution at
all. If you chose Choice (B), you may have misread Student 1 to say that the two
methods together account for 50% production.
Made by: Shahd A.Gaber

Question: 7
Correct Answer: J
Explanation:
J Student 2’s hypothesis is that seed distribution is the main reproductive strategy
for Taraxicum, while some incidental vegetative reproduction is possible. For this to be
true, the control field (experiment 1) would have the largest population, followed closely
by the field in experiment 3 (which has no vegetative reproduction), with only very few
dandelions in field 2 (which eliminated seed distribution). Choice (F) describes a
scenario where vegetative reproduction alone resulted in more dandelions than seed
distribution and vegetative reproduction together, which would be mysterious. Choice
(H), similarly, claims that the control experiment would have fewer offspring than the
experiment which eliminated vegetative reproduction. If the results in Choice (G) were
obtained, then removing dandelion’s seed distribution and vegetative reproduction
would have roughly equal effects, which would mean student 1 was correct.

(Test 19-1)
Question: 1
Correct Answer: B
Explanation:
B The transition from April to May shows a small increase in the number of reported
polio infections, and the transition from November to December shows a decrease, so
neither choices (C) nor (D) would be the correct answer. There are large increases in
the number of reported cases from January to February and from February to March,
but upon close inspection, the transition from February to March is larger. Therefore,
choice (B) is the best answer.

Question: 2
Correct Answer: J
Explanation:
J In June 2004, 80 cases of polio infection were reported, so 80 × 200 = 16,000 people
would have been at risk for contraction of the infection.

Question: 3
Correct Answer: A
Explanation:
A In all the Indian cities, there are more reported cases of polio virus infections in
August than in June. Choices (B) and (D) are consistent with decreases, not increases,
in the reported polio cases. The study described by Figure 2 only covers the dry
summer and rainy monsoon season, not the autumn or winter, so the explanation given
Made by: Shahd A.Gaber

in choice (C) is unsupported. Choice (A) is the best answer because the month of
August is expected to have more rainfall, and therefore is more likely to feature water
contamination.

Question: 4
Correct Answer: H
Explanation:
H Figure 2 presents findings that are applicable only to India, not Nigeria, so (F) is
incorrect. The findings only present information on reported infections during June and
August 2007, not on unreported infections (choice (G) is wrong) nor on winter months
(choice (J) is wrong). The findings do, however, show a stark contrast between the
small number of reported cases in the cities in western and southern India (Mumbai,
Chennai, and Hyderabad) and the large number of cases in the northern cities (New
Delhi and Kolkata), making (H) the best answer.

Question: 5
Correct Answer: D
Explanation:
D The passage states that the polio virus is most often transmitted through water
contaminated with human waste and makes no mention of the role (or the lack thereof)
that other life forms play in the transmission of the virus. The answer that summarizes
this is choice (D).

Question: 6
Correct Answer: H
Explanation:
H In June 2007, there were between 10 and 15 reported polio infections in Kolkata,
while in August 2007, there were between 20 and 25 reported polio infections. This is
an approximate doubling of the number of reported infections, so choice (H) is the best
answer.

(Test 20-2)
Question: 6
Correct Answer: B
Explanation:
B Examine the conditions associated with each group in Table 1. Group 6 is the only
group which does not include data from an area affected by habitat loss or declines in
prey population. It therefore allows the researchers to compare areas affected by these
factors to one unaffected by them. Choices (A), (C), and (D) suggest that Group 6
provides information to the researchers that is not included in the passage.
Made by: Shahd A.Gaber

Question: 7
Correct Answer: F
Explanation:
F Examine the conditions associated with Group 2. These areas have additional
seaweed which is consumed by marine mammals, the likely prey of the primarily
carnivorous polar bear. Choice (G) implies that polar bears may prefer to eat seaweed,
but there is nothing in the passage to support this statement. Choice (H) is eliminated
because the study does not directly measure the population density of prey animals.
Choice (J) would require additional information which links the population of prey
animals to Arctic sea ice.

Question: 8
Correct Answer: C
Explanation:
C A greater average population density ratio for a certain group indicates that more
polar bears are living in areas that are a part of that group. Choice (C) correctly lists the
groups by average population density ratio as given in Figure 1.

Question: 9
Correct Answer: G
Explanation:
G Group 1 exhibits declines in the number of marine mammals consumed by polar
bears. Choices (F) and (H) are not mammals so they are eliminated. Choice (J) is
eliminated because the population of polar bears is the outcome being studied as a
result of some other environmental change, and there is nothing in the passage to
suggest that polar bears consume other polar bears. Only choice (G) is a marine
mammal that is likely consumed by a polar bear.

Question: 10
Correct Answer: D
Explanation:
D You can use the process of elimination with the definitions in Table 1 to find the
correct answer for this problem. Choice (A) includes Groups 1 and 2; these are likely to
cause effects in opposite directions, not the same direction. Choices (B) and (C) each
include Group 4; as Group 4 is simply a combination of Groups 1 and 3, the researchers
are never directly comparing the effects of Group 4 conditions to any other conditions.
Choice (D) lists Groups 1 and 3, both groups where there have been conditions likely to
make it difficult for polar bears to survive. This is confirmed by data in Figure 1.
Made by: Shahd A.Gaber

Question: 11
Correct Answer: H
Explanation:
H Choices (F) and (J) are both true. As seen in Figure 1, average polar bear population
ratios for both Groups 1 and 3 are not equal to 1. If they were equal to 1, this would
indicate that there is no difference between those areas with the conditions listed and
those without. Choice (H) reverses the relationship between Groups 1 and 3 in Figure 1,
which means it is not supported. Choice (G) states the relationship from (H) correctly,
which means (G) is supported and therefore not the correct answer.

(Test 22-2)
Question: 7
Correct Answer: D
Explanation:
D Using Figure 1, determine the features of a Pipistrellus hesperus starting at Step 8
and work backwards. Step 8 describes it to have a forearm length < 40 mm. Step 7
describes it to have a tragus < 6 mm and curved. Step 5 describes it to have
a uropatagium not heavily furred, and Step 1 describes it to have ears shorter than 25
mm. Only choice (D) refers to a feature of Bat IV, uropatagium heavily furred, that
differs from those of Pipistrellus hesperus found using the above method. Alternatively,
you could use the features of bat IV from Table 1 and find the point on Figure 1 where
the result differs from the path necessary to get to Pipistrellus hesperus.

Question: 8
Correct Answer: G
Explanation:
G Follow Figure 1 step by step using the descriptions for bats I and II from Table 1 until
you find the last step with the same result. Starting at Step 1, both bats have
ears shorter than 25 mm making Step 5 next. Therefore, choice (F) must be wrong.
Both bats have a uropatagium that is not heavily furred, making Step 7 next. Bat I has
a 4 mm, curved tragus, and Bat II has a 7 mm, straight tragus. Therefore, the results of
Step 7 differ making choices (H) and (J) wrong. This leaves choice (G), Step 5, as the
last point where the bats had similar traits. Note that the obvious fringe of fur on the
uropatagium of Bat II does not come into play until Step 9 of Figure 1.
Made by: Shahd A.Gaber

Question: 9
Correct Answer: A
Explanation:
A All of the choices are in the kingdom Animalia and phylum Chordata. Vesper bats, like
all bats, belong to the class Mammalia. Mammals are vertebrates with sweat glands,
hair, and similar middle ear structures that give birth to live young (except monotremes
which lay eggs).

Question: 10
Correct Answer: G
Explanation:
G Both species are found at Step 6 in Figure 1. Step 6 can only be reached from
having heavily furred uropatagium in Step 5, eliminating choice (H). Step 5 can only be
reached from having shorter than 25 mm ears in Step 1, eliminating choice (J). Choice
(F) refers to Step 3 which is not part of the path for either species, meaning it may or
may not be a common trait. Choice (G) is more definitive because it refers to the only
feature that neither species can have according to Figure 1. It is not possible to get to
Step 6 if the ears are greater than 25 mm long.

Question: 11
Correct Answer: D
Explanation:
D Using the features listed in Table 1 for Bat II, follow the steps in Figure 1. Bat II's 18
mm long ears lead from Step 1 to Step 5 in Figure 1. The uropatagium overall is not
heavily furred, which then leads to Step 7. The tragus is 7 mm and straight, leading next
to Step 9. Since there is an obvious fringe of fur on the edge of the uropatgium, bat II
is Myotis thysanodes. Choice (D), Myotis volans, is most likely the closest genetic
relative because it is in the same genus, has very similar features, and is adjacent on
Figure 1.

(Test 22-4)
Question: 17
Correct Answer: C
Explanation:
C Pepsin is described as an enzyme that is involved with protein breakdown, and that is
active in an acidic environment. Of the choices listed, only choice (C) is a component of
the digestive system, which is responsible for the breakdown of nutrients. Also, the
stomach is an organ with a highly acidic environment.
Made by: Shahd A.Gaber

Question: 18
Correct Answer: F
Explanation:
F Note where the Pepsin Activity in Table 2 is High. There is no evidence on the table
that Pepsin activity is high at any pH higher than 3.5, so you can only be sure of choice
(F).

Question: 19
Correct Answer: C
Explanation:
C Pepsin is capable of high activity in the absence of anserine in Trial 4, thus choice (A)
cannot be correct. Pepsin activity is also high in the presence of anserine in Trial 3, thus
choice (B) cannot be correct. Casein is described as a protein that can be digested by
pepsin, so choice (D) cannot be correct. This leaves choice (C) as the only possible
answer. Another way to approach this problem is to notice that what makes Trial 5
different from Trials 3 and 4 is that it does not contain casein. If no pepsin activity is
seen when casein is absent, it would follow that casein is a substance that can be
digested by pepsin, supporting choice (C).

Question: 20
Correct Answer: H
Explanation:
H In order for casein to remain undigested, casein must first be present in the solution.
Trials 5 and 6 do not contain casein, so choices (G) and (J) can be eliminated. Choice
(H) is a better choice than choice (F) because the high pepsin activity in Trials 3 and 4
would break casein down into the smaller peptides.

Question: 21
Correct Answer: A
Explanation:
A Trial 3 in Experiment 1 is conducted at a pH of 3.0 and at a temperature of 40°C.
While all of answer choices feature Experiment 2 trials conducted at a temperature of
40°C, only Trial 9 is conducted at a similar pH of 3.0. Therefore, choice (A) is the best
answer.

Question: 22
Correct Answer: G
Explanation:
G The results from Experiment 1 show high activity of pepsin, meaning a fast rate of
protein digestion by pepsin, at a temperature of 40°C, which excludes choices (C) and
Made by: Shahd A.Gaber

(D). The results from Experiment 2 show high activity of pepsin at pH values that are
less than 4.0, so choice (B) is the best answer.

(Test 23-4)
Question: 17
Correct Answer: D
Explanation:
D According to Figure 1, the greatest reflectance for blue-green algae occurs around
550 nm. According to Table 1, a wavelength of 550 nm is associated with the color
green, making choice (D) correct.

Question: 18
Correct Answer: H
Explanation:
H Photosynthesis is the chemical reaction that identifies autotrophic organisms, which
make their own food, specifically sugar. Binary fission is the asexual reproduction of
bacteria. Condensation is the transition of water from gas to liquid. Respiration is the
opposite reaction of photosynthesis, breaking down glucose rather than making it.

Question: 19
Correct Answer: A
Explanation:
A Choice (A), 400 nm, is the only choice where the green algae curve is higher than the
diatoms curve. For all of the other choices, the diatoms curve is higher than the green
algae curve, so the relative reflectance is higher for diatoms.

Question: 20
Correct Answer: J
Explanation:
J Protists are organisms which cannot be classified as plants or animals. They can be
one or many celled, but are always simple in construction. Choice (J) is correct because
all algae are protists.

Question: 21
Correct Answer: B
Explanation:
B The first paragraph of the passage tells you that as the amount of algae in a water
sample increases, the water sample reflects light more like the algae so you know that
Made by: Shahd A.Gaber

all you need to do with this question is look for the algae that reflects light like the water
sample. The best way to figure out this question is to quickly locate the high and low
points on the water sample graph. The water sample has peaks at 550 nm and 0.08
reflectance and at 700 nm and 0.07 reflectance. The water sample has low points at
350 nm and 0.03 reflectance and at about 660 nm and 0.04 reflectance. Now, look at
the algae graph. Once you match those points to the algae graph, it is easy to see the
choice (B), diatoms, is the correct answer.

(Test 23-6)
Question: 28
Correct Answer: J
Explanation:
J According to Table 1, only the haloarchaeal and bacterial cells show + signs for either
acid or CO2 presence under green light. Therefore, choice (J) is the best answer.

Question: 29
Correct Answer: B
Explanation:
B A plant cell alone produces only CO2 and absorbs most of the light (i.e., has low
transmittance) in the presence of red light. A haloarchaeal cell alone produces only acid
and absorbs most of the light (i.e., has low transmittance) in the presence of green light.
Since these two cell types do not interfere with each other, they will most likely continue
to have production of CO2 and low transmittance in the presence of red light, as well as
the production of acid and low transmittance in the presence of green light. Choice (B)
summarizes this.

Question: 30
Correct Answer: G
Explanation:
G Only the plant and bacterium produce CO2 in the presence of red light, so you can
immediately eliminate choices (F) and (H). Then, under the green light, the bacterium
produces both acid and CO2 and the plant produces neither, making choice (G) the best
answer.
Made by: Shahd A.Gaber

Question: 31
Correct Answer: C
Explanation:
C According to Experiment 1, the production of acid is a sign of growth. According to
Experiment 2, low transmittance indicates that light is being absorbed by pigments to
generate energy. The evidence that haloarchaea use light to generate energy and grow
must include production of acid and low transmittance. These are both seen only in the
presence of green light, so choice (C) is the best answer.

Question: 32
Correct Answer: G
Explanation:
G In red light, the plant cell Rosa carolina will produce CO2 but not acid. When CO2 is
present, a gas bubble appears above the solution. If acid is not present, then the
solution appears colored. The illustration in choice (G) best represents this.

Question: 33
Correct Answer: C
Explanation:
C The results of Experiment 1 show that bacteria and haloarchaea produce different
products in the presence of red and green light. The passage does not mention any
other relationship between haloarchaea and bacteria. Therefore, the best answer is
choice (C): The two types of cells show different growth patterns, so we cannot
conclude that they are closely related.

(Test 23-7)
Question: 34
Correct Answer: G
Explanation:
G The second sentence of the last paragraph for the 3-Domain Hypothesis implies that
the more different the genetic sequence of rRNA, the farther back in time two groups of
organisms diverged or split on the evolutionary tree. Similar genetic sequences would
imply more closely related species, eliminating choice (F). A relationship between ester
or ether linkages and divergence is not discussed in the passage, eliminating choices
(H) and (J).
Made by: Shahd A.Gaber

Question: 35
Correct Answer: D
Explanation:
D Phospholipids have membranes but are not organelles themselves, eliminating
choice (A). Ribosomes are not described as membrane-bound organelles in the
passage, and the introduction implies that the Archaea have ribosomes because they
have rRNA, eliminating choices (B) and (C). Nuclei are membrane-bound organelles
found only in eukaryotes.

Question: 36
Correct Answer: J
Explanation:
J The last paragraph states that eukaryotes and prokaryotes have ester linkages in their
cell membranes, eliminating choice (F). The introduction defines the difference between
eukaryotes and prokaryotes as the presence or absence of membrane-bound
organelles, eliminating choice (G). Prokaryotes reproduce asexually as mentioned in the
2-Domain Hypothesis, eliminating choice (H). All of the organisms described in the
passage are composed of cells.

Question: 37
Correct Answer: C
Explanation:
C The scientist supporting the 2-Domain Hypothesis opens the argument by defining
the Archaea as prokaryotes. The introduction states the Archaea contain rRNA,
eliminating choice (A). Although ether linkages are found in the Archaea, this is an
argument proposed by the scientist supporting the 3-Domain Hypothesis, eliminating
choice (B). Protein synthesis can occur in the cytoplasm of all living organisms, and is
not mentioned as a defining characteristic of any of the domains in the passage,
eliminating choice (D).

Question: 38
Correct Answer: F
Explanation:
F The observation of cellular metabolism similar to that found in eukaryotes discredits
the arguments mentioned in the 2-Domain Hypothesis, eliminating choices (H) and (J).
Since the metabolic process is similar to eukaryotes, choice (G) is also eliminated.
Made by: Shahd A.Gaber

Question: 39
Correct Answer: C
Explanation:
C Neither scientist makes the claim that the Archaea have membrane-bound
organelles, eliminating choice (A). Microscopes play a vital role in accurately describing
organisms, and this is not mentioned one way or the other in the passage, eliminating
choice (B). The distance in relationship between eukaryotes and the Archaea is not the
primary argument of each scientist. They are arguing more about whether to break up
prokaryotes into bacteria and the Archaea, eliminating choice (D). The fact that the
Archaea have ether linkages instead of ester linkages is mentioned by the scientist
arguing for the 3-Domain Hypothesis as a significantly distinguishing characteristic,
which allows the Archaea to occupy harsh environments.

Question: 40
Correct Answer: G
Explanation:
G Phospholipids are described in the passage as having a water-soluble subunit and a
water-insoluble subunit. Therefore, it makes the most sense that the water-soluble
subunits (circles) would arrange in a manner such that they were exposed to water
while keeping the water-insoluble subunits (lines) away from the water.

(Test 24)
Question: 1
Correct Answer: D
Explanation:
(D) The endosperm may be yellow or white. Since the endosperm is located underneath
the aleurone layer, its color is only visible when the aluerone is colorless. A purple or
red aluerone masks the endosperm.

Question: 2
Correct Answer: A
Explanation:
(A) The pericap is the outermost layer of the corn seed, as shown in Figure 1.1. Since
no color phenotypes related to the pericap are mentioned, it can be inferred that the
pericap is colorless, allowing the color phenotype of the aleurone and endosperm to be
visible.
Made by: Shahd A.Gaber

Question: 3
Correct Answer: C
Explanation:
(C) Table 1.1 lists three alleles (C', C, and c) for the aleurone color inhibitor trait. The
other traits listed each have two alleles (Y or y, R or r, and P or p).

Question: 4
Correct Answer: B
Explanation:
(B) The aleurone may have a purple, red, or colorless phenotype. Colorless is not itself
a kernel color phenotype because a colorless aleurone allows the endosperm color,
either yellow or white, to be visible. Thus, there are 4 possible unique kernel color
phenotypes (purple, red, yellow, and white).

Question: 5
Correct Answer: A
Explanation:
(A) The endosperm color genotype yy produces a white kernel when the aleurone
phenotype is colorless. According to Table 1.1, the aleurone color genotype rr and the
aleurone color inhibitor genotype C'c both produce a colorless aleurone, and the
aleurone color modifier genotype PP has no effect on aleurone color.

Question: 6
Correct Answer: C
Explanation:
(C) The aleurone color modifier genotype pp changes a purple kernel to red. A purple
kernel results from a genotype that includes at least one R and one C. RRCCppyy is the
only answer choice that satisfies these conditions. The endosperm color genotype (yy)
does not matter in this case because the aleurone color masks it.

Question: 7
Correct Answer: B
Explanation:
(B) The notation P > p in the Allele Relationships column of Table 1.1 indicates that the
allele P is dominant to the allele p, and thus the phenotype of p will be masked
when P is present. This is supported by the fact that the genotypes PP and Pp both
produce the same phenotypic outcome, while pp produces a different phenotypic
outcome.
Made by: Shahd A.Gaber

Question: 8
Correct Answer: D
Explanation:
(D) Each kernel is a separate seed, or offspring, produced from the same pair of
parents. Each seed gets the same genes, but the two specific alleles of each gene that
each seed receives can be different.

Question: 9
Correct Answer: A
Explanation:
(A) According to Table 1.1, a white phenotype can only result from a genotype
containing the endosperm color combination yy. rrCCppyy is the only genotype listed
that corresponds to a white phenotype.

Question: 10
Correct Answer: B
Explanation:
(B) To have a yellow color, a kernel must have a colorless aleurone. Since both parents
are red (and based on the relationships information in Table 1.1), the only genotype
information known for certain about the kernels in Cross 2 is that each must have
received the aleurone color modifier alleles pp from the parents. It is possible that the
offspring kernels could receive from the parents some combination of R and C alleles
that would produce a colorless aleurone, allowing the yellow endosperm to be visible.

Question: 11
Correct Answer: C
Explanation:
(C) The ratio can be found by dividing the number of red kernels by the number of
yellow kernels. Using the data from Ear 1, 381 red kernels ÷ 126 yellow kernels =
approximately 3. This means that the ratio of red to yellow is 3 to 1, or 3:1. This same
ratio can be found using the data from Ear 2 or 3, or by combining all of the data from
Cross 2.

Question: 12
Correct Answer: A
Explanation:
(A) Cross 1 produced 100% yellow kernels. Repeating this cross will provide the
greatest chance of producing more yellow kernels. Since Crosses 2 and 3 both
produced lower percentages of yellow kernels, repeating either or both crosses would
lower the overall probability of producing yellow kernels.
Made by: Shahd A.Gaber

Question: 14
Correct Answer: C
Explanation:
(C) Because yellow and white are two different alleles of the same gene (endosperm
color), substituting a white parent for the yellow parent in Cross 3 would produce an ear
with white kernels instead of yellow. Since endosperm color is masked by aleurone
color, the kernels exhibiting purple aleurone color would not be affected by a change in
endosperm color.

Question: 15
Correct Answer: C
Explanation:
(C) The passage presents corn seed color as a trait that is influenced by four different
genes whose influences are listed in Table 1.1. The passage does not mention an
ability to change phenotype over time or any other functions of genes besides the
influence on seed color.

(Test 26)
Question: 1
Correct Answer: A
Explanation:
(A) As illustrated in Figure 1.3, gram-positive bacteria have a cell wall composed of a
thick layer of peptidoglycan and no outer membrane surrounding it. Gram-negative
bacteria have a much thinner layer of peptidoglycan, surrounded by an outer membrane
consisting of a lipid bilaye

Question: 2
Correct Answer: C
Explanation:
(C) As illustrated in Figure 1.3, both gram-positive and gram-negative cells have a
periplasmic space located between the peptidoglycan layer and the cell membrane
below.

Question: 3
Correct Answer: B
Explanation:
(B) The lipid bilayer that makes up the outer membrane of gram-negative cells consists
of an outer lipopolysaccharide layer and an inner phospholipid layer. Porins are
transport channels that exist within this lipid bilayer, but they do not extend into the
peptidoglycan layer. The thick peptidoglycan layer of gram-positive cells has no porins.
Made by: Shahd A.Gaber

This suggests that peptidoglycan is more permeable than the lipopolysaccharide-


phospholipid bilayer, because the peptidoglycan layer does not contain any special
transport channels, and both cell types must have the ability to transport substances
into and out of the cell.

Question: 4
Correct Answer: D
Explanation:
(D) As explained in the passage, ethyl alcohol is used as a decolorizer, not as a stain.
Crystal violet is the primary stain, and safranine and fuchsin are two common
counterstains, used in Gram staining.

Question: 5
Correct Answer: D
Explanation:
(D) At the end of the Gram staining technique, cells appear either purple or red under
the microscope. Gram-positive cells appear purple as a result of retaining the crystal
violet stain. Gram-negative cells appear red because they do not retain the crystal violet
and are subsequently stained by the counterstain.

Question: 6
Correct Answer: C
Explanation:
(C) The purpose of adding iodine in the Gram staining technique is to form a complex
with crystal violet that will become trapped inside gram-positive cells during the
decolorization step. Application of the primary stain, crystal violet, must occur before
iodine is added to allow this complex to form.

Question: 7
Correct Answer: D
Explanation:
(D) Since gram-negative cells appear colorless after the decolorization step washes the
primary stain out, the addition of a counterstain allows these cells to be seen more
easily under a microscope. As noted in the passage, the red counterstain is lighter
colored than the purple primary stain, so the addition of the counterstain does not affect
the appearance of gram-positive cells.
Made by: Shahd A.Gaber

Question: 8
Correct Answer: A
Explanation:
(A) Ethyl alcohol acts as a decolorizer in the Gram staining technique. As described in
the passage, the decolorization step degrades the outer membrane of gram-negative
bacteria. Since this outer membrane consists of a lipid bilayer, it is most reasonable to
assume that a chain of lipids will degrade in the presence of ethyl alcohol.

Question: 9
Correct Answer: A
Explanation:
(A) According to the dichotomous key in Table 1.6, Streptococcus bacteria are gram-
positive. Gram-positive bacteria appear purple when exposed to Gram staining.

Question: 10
Correct Answer: A
Explanation:
(A) Step 1a directs the user to continue to step 2 for gram-positive cells. Step 1b directs
the user to continue to step 3 for gram-negative cells. Since the descriptions used in
both steps 2 and 3 are identical, it can be assumed that rod-shaped and sphere-shaped
bacteria species exist in both groups.

Question: 11
Correct Answer: B
Explanation:
(B) Two of the genera (Staphylococcus and Streptococcus) in Table 1.6 are identified
as gram-positive. Gram-positive bacteria have a thick peptidoglycan layer.

Question: 12
Correct Answer: A
Explanation:
(A) By following Table 1.6 beginning at step 1, it can be determined that the
genus Pseudomonas consists of bacteria that are gram-negative (step 1b), rod-shaped
(step 3a), and do not ferment lactose (step 5b).

Question: 13
Correct Answer: D
Explanation:
(D) By working backward through Table 1.6, it can be determined that
both Pseudomonas and Enterobacter are gram-negative (step 1b) and rod-shaped (step
Made by: Shahd A.Gaber

3a). Pseudomonas is identified as not fermenting lactose in step 5b. Enterobacter is


identified as fermenting lactose (step 5a) and using citric acid (step 6a).

Question: 14
Correct Answer: B
Explanation:
(B) Based on Table 1.6, the genus Escherichia has gram-negative cells, which should
appear red after Gram staining because they absorb the counterstain instead of
retaining the primary stain. This means that colorless gram-negative cells most likely
indicate an error in the counterstaining step.

Question: 15
Correct Answer: A
Explanation:
(A) According to the information in Table 1.6, Staphylococcus and Streptococcus are
both gram-positive bacteria. Since gram-positive bacteria are less resistant than gram-
negative bacteria to antibiotic treatment, it would be most effective to treat infections
caused by bacteria in these genera with penicillin.

(Test 27)
Question: 1
Correct Answer: C
Explanation:
(C) According to the information in Table 2.1, European honeybee and jack jumper ant
males are both haploid. The male swamp wallaby has one less chromosome than the
female. Slime mold has no notation indicating a variability in chromosome numbers
among individuals of the species.

Question: 2
Correct Answer: D
Explanation:
(D) The coyote (2n = 78 chromosomes) and dog (2n = 78) both belong to the
genus Canis. The horse (2n = 64) and the donkey (2n = 62) both belong to the
genus Equus. The Bengal fox (2n = 60 chromosomes) and red fox (2n = 34) both
belong to the genus Vulpes. These pairs of species demonstrate that two members of
the same genus may have the same chromosome count (coyote and dog) or very
different chromosome counts (Bengal fox and red fox).
Made by: Shahd A.Gaber

Question: 3
Correct Answer: B
Explanation:
(B) As the diploid number of chromosomes decreases, the complexity of the organism
does not decrease, as exemplified by a human having fewer chromosomes than a
potato. Neither does the complexity of the organism increase, as exemplified by oats
having fewer chromosomes than a human. Therefore, based on the information in Table
2.1, no significant correlation can be identified between the complexity of organisms and
their diploid chromosome count.

Question: 4
Correct Answer: B
Explanation:
(B) The adder's-tongue fern has the greatest diploid number of chromosomes (2n =
1,260) in Table 2.1. A fern is a plant, which belongs to the kingdom Plantae.

Question: 5
Correct Answer: A
Explanation:
(A) The dog has 78 diploid chromosomes, while a human has 46. The
silkworm, Bombyxmori, falls between these numbers with a total of 54.

Question: 6
Correct Answer: B
Explanation:
(B) The zebra fish and pineapple both have 50 diploid chromosomes. This means that
50 total chromosomes are contained in the nucleus of each somatic cell of both
organisms.

Question: 7
Correct Answer: D
Explanation:
(D) The data in Table 2.1 indicates that a wide variability of chromosome counts exists,
even among related species. Because of this, the diploid chromosome count of a new
species cannot be predicted easily without more information. Genetic testing such as
karyotyping can be used to positively identify the number of chromosomes for each
species.
Made by: Shahd A.Gaber

Question: 8
Correct Answer: C
Explanation:
(C) Gametes contain the haploid (n) number of chromosomes. Table 2.1 identifies the
diploid (2n) number of chromosomes that are present in an organism's somatic cells.
The horse (Equus ferus caballus) has a diploid number of 2n = 64. This means the
haploid number in gametes would be n = 32.

Question: 9
Correct Answer: C
Explanation:
(C) Table 2.1 indicates that male European honeybees are haploid rather than diploid.
This means a male has n chromosomes, while a female has 2n chromosomes. Since
the diploid number of chromosomes is 32, n = 16. Therefore, females have 16 more
total chromosomes than males.

Question: 10
Correct Answer: D
Explanation:
(D) The total number of chromosomes in a somatic cell is referred to as the diploid
number. The diploid number is represented by the term 2n, in which n represents the
number of chromosomes in a gamete from the same organism. The number of
chromosomes in a somatic cell is twice the number of chromosomes in a gamete.

Question: 11
Correct Answer: C
Explanation:
(C) As stated in the passage, the gametes of most species contain one complete set of
chromosomes, and the somatic cells contain two complete sets. In polyploid species,
the somatic cells contain more than two sets of chromosomes, but the gametes still
contain half of the total number of chromosomes in a somatic cell. This means a gamete
will contain more than one complete set of chromosomes.

Question: 12
Correct Answer: A
Explanation:
(A) Table 2.1 indicates that alfalfa is a tetraploid species. The prefix tetra means "four."
Therefore, as described in the passage, the somatic cells of a tetraploid species have
four complete sets of chromosomes.
Made by: Shahd A.Gaber

Question: 13
Correct Answer: B
Explanation:
(B) The monoploid number (x) identifies the number of chromosomes an organism has
in one set. For the hexaploid oat species Avena sativa, 6x = 42. This means x = 7, or
the number of chromosomes present in one set is 7.

Question: 14
Correct Answer: A
Explanation:
(A) Table 2.1 indicates that the potato (Solanum tuberosum) is a tetraploid. Tetraploid
organisms have four complete sets of chromosomes. As explained in the passage, the
diploid number (2n) is always twice the haploid number (n). 2n always represents the
total number of chromosomes in a somatic cell, regardless of whether the organism
exhibits any type of polyploidy. Since a tetraploid has four complete sets of
chromosomes, the total number of chromosomes (2n) must be divided by 4 to
determine the monoploid number (x).

(Test 28)
Question: 1
Correct Answer: B
Explanation:
(B) According to Scientist 1, increased temperatures will help some crops and harm
others. The growth rate for many crops increases with temperature, which is beneficial
to those crops and the agriculture industry. However, increased temperature
suppresses the reproductive ability of some crops, which endangers their survival.

Question: 2
Correct Answer: D
Explanation:
(D) Based on the opinion of Scientist 1, a crop has its greatest yield close to its optimal
growth temperature. Since average temperatures are predicted to rise as a result of
climate change, plants with higher optimal growth temperatures are more likely to
produce higher yields than those with lower optimal growth temperatures because they
can better withstand the heat.
Made by: Shahd A.Gaber

Question: 3
Correct Answer: D
Explanation:
(D) Scientist 2 discussed the potential for crop destruction due to an increase in the
frequency of flooding. Scientist 1 did not discuss the effects of flooding or any other
extreme weather events.

Question: 4
Correct Answer: C
Explanation:
(C) Scientist 2 discussed an increase in the production of weeds, pests, and fungi in
response to the effects of climate change. Only Scientist 1 addressed the potential
effect of climate change on seed production.

Question: 5
Correct Answer: A
Explanation:
(A) In Scientist 1's opinion, increases in average temperature and carbon dioxide levels
will have positive effects on some types of crops and negative effects on others.
Scientist 2 only discusses the negative effects of these increases.

Question: 6
Correct Answer: B
Explanation:
(B) Scientist 2 states that increased carbon dioxide levels will allow weeds, pests, and
fungi to thrive. The increased presence of these organisms is predicted to impact crop
yields negatively by competing with and damaging crops.

Question: 7
Correct Answer: B
Explanation:
(B) In Scientist 2's opinion, climate change will lead to an increase in agricultural pests.
An observed increase in pesticide use would provide evidence to support this opinion.
Made by: Shahd A.Gaber

Question: 8
Correct Answer: B
Explanation:
(B) Scientist 1 states that once the optimal growth temperature is surpassed, crop yields
will decline. Though crops will still grow in the higher-than-optimal temperatures, the
amount of growth will be reduced.

Question: 9
Correct Answer: D
Explanation:
(D) Both scientists agree that increasing average temperatures will be beneficial to
some organisms, though they do not agree on the particular organisms that will benefit.
Scientist 1 suggests that increasing average temperatures will increase the growth rate
of many crops, whereas Scientist 2 suggests that they will increase the growth of
weeds, pests, and fungi, which will inhibit the growth of crops.

Question: 10
Correct Answer: D
Explanation:
(D) Scientist 1 states that increased carbon dioxide levels will increase crop yields. It
can be inferred that this change would be due to an increase in the rate at which crops
carry out photosynthesis, since increased food production would lead to increased
growth.

Question: 11
Correct Answer: A
Explanation:
(A) Scientist 2 discusses the northward spread of weeds and pests as being detrimental
to northern crops. While pests will directly damage crops, weeds will compete with
crops for resources such as soil nutrients and water.

Question: 12
Correct Answer: D
Explanation:
(D) According to Scientist 1, higher average temperatures and carbon dioxide levels will
have direct, positive effects on many crops. This hypothesis can best be tested by
gathering data on these two variables and the crop yields that result.
Made by: Shahd A.Gaber

Question: 13
Correct Answer: A
Explanation:
(A) According to Scientist 2, increased temperatures and carbon dioxide levels will
cause the ranges of more heat-tolerant southern species to expand northward. As
southern species expand northward, the ranges of northern species will likely narrow
due to the increase in competition and new pests.

Question: 14
Correct Answer: B
Explanation:
(B) Scientist 1 states that an increase in carbon dioxide causes crop yields to increase.
However, once the optimal growth temperature for a crop is surpassed, crop yields will
decline. Graph B best matches the scientist's description, with the maximum of Graph C
representing the point at which the optimal growth temperature has been reached.

(Test 29)
Question: 1
Correct Answer: B
Explanation:
(B) According to the photosynthesis equation, plants take in carbon from the
environment in the form of carbon dioxide. This atmospheric carbon is used to produce
organic carbon in the form of carbohydrates like glucose.

Question: 2
Correct Answer: D
Explanation:
(D) According to the photosynthesis equation, plants use sunlight to convert carbon
dioxide and water into a useable form of carbon (glucose). The by-product of this
process is oxygen, which is released back into the environment.

Question: 3
Correct Answer: B
Explanation:
(B) The passage states that each leaf was initially exposed to a light intensity of 300
μE/m2/s for a period of time to stimulate photosynthesis. According to Figure 2.5, this
initial 300 μE/m2/s exposure began at approximately 5 minutes and ended at
approximately 25 minutes, for a total duration of 20 minutes.
Made by: Shahd A.Gaber

Question: 4
Correct Answer: A
Explanation:
(A) As shown in Figure 2.7, the air temperature increased continuously over the course
of the study. This continuous increase can best be attributed to the increase in light
intensity over the same time. It can be inferred that increasing the intensity output of a
light source also increases the heat output.

Question: 5
Correct Answer: A
Explanation:
(A) The reactants of the photosynthesis equation are carbon dioxide and water.
Figure 2.6 depicts the change in carbon dioxide concentration as recorded by
sensors within the chamber over the course of the study.

Question: 6
Correct Answer: C
Explanation:
(C) According to the passage, students manipulated the light intensity within the
chamber and observed the effects on carbon dioxide concentration. This means that
light intensity, depicted in Figure 2.5, is the independent variable.

Question: 7
Correct Answer: D
Explanation:
(D) The lower the concentration measured within the chamber, the faster the leaf is
absorbing carbon dioxide. This means that the lowest point on the graph in Figure 2.6,
occurring at approximately 50 minutes, represents the highest absorption rate.
According to Figure 2.5, the leaf is being exposed to a light intensity of 1,000 μE/m 2/s at
50 minutes.

Question: 8
Correct Answer: B
Explanation:
(B) In Table 2.2, the mean carbon dioxide exchange rate was the highest for the
sunflower plant as compare to the other three plant types. This means that sunflower
leaves absorb carbon dioxide fastest. Since carbon dioxide and oxygen are exchanged
in a 1:1 ratio, it can also be stated that sunflower leaves release oxygen fastest.
Made by: Shahd A.Gaber

Question: 9
Correct Answer: C
Explanation:
(C) According to Table 2.2, rhoeo leaves showed the smallest standard deviation (1.3)
across 10 trials of all four plant species studied. A low standard deviation indicates a
low amount of variability across trials.

Question: 10
Correct Answer: C
Explanation:
(C) The passage states that carbon dioxide and oxygen are exchanged in a 1:1 ratio
during photosynthesis. As the carbon dioxide concentration in the chamber decreases,
the oxygen concentration increases in direct proportion. Therefore, the oxygen
concentration graph, just like the light intensity graph in Figure 2.5, would be the inverse
of the carbon dioxide concentration graph in Figure 2.6.

Question: 11
Correct Answer: B
Explanation:
(B) In Table 2.2, the mean carbon dioxide exchange rate for pothos leaves is 6.0
μmol/m2/s. The means for both sunflower leaves (17.4 μmol/m 2/s) and water hyacinth
leaves (14.3 μmol/m2/s) are greater. A greater carbon dioxide exchange rate indicates a
greater rate of photosynthesis.

Question: 12
Correct Answer: C
Explanation:
(C) Figures 2.5 and 2.6 indicate that light intensity and carbon dioxide concentration
within the chamber show an inverse correlation. This means that light intensity directly
correlates to carbon dioxide absorption and therefore photosynthesis rate.

Question: 13
Correct Answer: A
Explanation:
(A) In Table 2.2, the highest carbon dioxide exchange rate was recorded for the
sunflower leaf (27 μmol/m2/s) during Trial 3. A higher value indicates a faster exchange
rate.
Made by: Shahd A.Gaber

Question: 14
Correct Answer: A
Explanation:
(A) As indicated in Figures 2.5 and 2.6, a plant's rate of photosynthesis depends on the
light intensity in its environment. Plants with a low maximum rate of photosynthesis
have a low need for light, making them more tolerant to low-light environments. In Table
2.2, the mean carbon dioxide exchange rate was observed to be the lowest for rhoeo
leaves, at 3.5 μE/m2/s. A low exchange rate indicates a low rate of photosynthesis.

Question: 15
Correct Answer: D
Explanation:
(D) Figures 2.5 and 2.6 indicate carbon dioxide concentration within the chamber is
lowest when light intensity is highest. This means that the maximum carbon dioxide
exchange rates identified in Table 2.2 were most likely recorded at 50 minutes, when
light intensity was at its maximum (1,000 μE/m2/s) and carbon dioxide concentration
was at its minimum (approximately 880 ppm).

(Test 30)
Question: 1
Correct Answer: D
Explanation:
(D) Four possible ABO blood types (A, B, AB, and O) are listed in Table 3.1, and two
possible Rh blood types (Rh+ or Rh–) are listed in Table 3.2. Each of the ABO blood
types may be paired with either of the Rh blood types, resulting in eight possible
medical blood types.

Question: 2
Correct Answer: B
Explanation:
(B) As seen in Table 3.1, each ABO blood type is named for the antigen(s) present on
the red blood cells. Blood type A has A antigens, and so on. Type O is so named
because it has no antigens.
Made by: Shahd A.Gaber

Question: 3
Correct Answer: A
Explanation:
(A) According to the passage, a cell's antigens serve as a unique chemical signature
that distinguishes it from other cell types. This allows the body to determine the identity
of a wide variety of different cells.

Question: 4
Correct Answer: A
Explanation:
(A) As seen in Table 3.2, blood identified as Rh+ contains the Rh antigen but not the Rh
antibodies.

Question: 5
Correct Answer: B
Explanation:
(B) Blood type is identified by the antigens present on the red blood cells. Since only B
antigens are present, the blood type would be B–. The presence of anti-A and anti-Rh
antibodies means that A and Rh antigens are absent.

Question: 6
Correct Answer: C
Explanation:
(C) Figure 3.1 indicates that 9% of the general population has type B+ blood and 2%
has type B– blood, for a total of 11% of the general population with type B blood.

Question: 7
Correct Answer: D
Explanation:
(D) According to Figure 3.1, only 1% of the general population has the AB– blood type.

Question: 8
Correct Answer: A
Explanation:
(A) A higher percentage of individuals of Asian ethnicity have B+ blood (25%) than any
other ethnicity. It can therefore be inferred that this blood type is more common in Asia
than in the other three continents.
Made by: Shahd A.Gaber

Question: 9
Correct Answer: C
Explanation:
(C) A antigens are present on the red blood cells of both type A and type AB blood. The
sum of type A+ (33%), type A– (6%), type AB+ (4%), and type AB– (1%) is 44%.

Question: 10
Correct Answer: B
Explanation:
(B) In Table 3.3, 53% of Hispanic individuals, or a little over half, have O+ blood. The
percentage of the general population that is Hispanic and has this blood type cannot be
determined from the data provided.

Question: 11
Correct Answer: A
Explanation:
(A) Based on the data in Table 3.3, an individual of African-American ethnicity has an
18% chance of having B+ blood. This is greater than the 9% of the general population
with the same blood type.

Question: 12
Correct Answer: C
Explanation:
(C) As illustrated in Table 3.1, AB blood contains A and B antigens but no antibodies.
Since no antibodies are present, the antigens on the donated blood will not be attacked,
leading to no immune reaction to the donated type A blood.

Question: 13
Correct Answer: B
Explanation:
(B) As illustrated in Tables 3.1 and 3.2, blood type O– has no antigens on its red blood
cells. If no antigens are present, there is nothing to trigger antibodies to attack.

Question: 14
Correct Answer: C
Explanation:
(C) An individual can safely receive a transfusion of his or her own blood type because
the donor blood contains the exact same antigens (A antigens in this case) as are
already present, triggering no antibody attack. Any individual can also receive a
transfusion of O– blood because it is the universal donor, containing no antigens at all.
Made by: Shahd A.Gaber

Question: 15
Correct Answer: C
Explanation:
(C) According to Table 3.3, 1% of the Caucasian population has AB– blood. Figure 3.1
indicates that 1% of the general population also has this blood type.

(Test 31)
Question: 1
Correct Answer: A
Explanation:
(A) According to the passage, a DNA ladder is loaded into the first well in the agarose
gel. The DNA samples being studied are loaded in the subsequent wells.

Question: 2
Correct Answer: D
Explanation:
(D) In Figure 3.2, the cathode is shown at the top of the diagram and the anode is at the
bottom. When the power supply is turned on, the resulting electric current causes the
DNA samples loaded in the wells to travel away from the negative charge produced by
the cathode and toward the positive charge of the anode.

Question: 3
Correct Answer: B
Explanation:
(B) A DNA ladder is a solution containing DNA fragments of known sizes. When an
electrophoresis procedure is run, the migration of these fragments provides a reference
by which to estimate the sizes of DNA fragments in the samples.

Question: 4
Correct Answer: A
Explanation:
(A) Since Table 3.4 identifies the voltage range as 0.25–7 V/cm, a voltage of 6 V/cm is
relatively high. Table 3.4 indicates that a high voltage may cause smearing or poor
resolution of large DNA fragments.
Made by: Shahd A.Gaber

Question: 5
Correct Answer: C
Explanation:
(C) In Table 3.5, a 0.5% agarose concentration is recommended to resolve DNA
fragments of 1–30 kb, or a 29 kb range. This range decreases incrementally as agarose
concentration increases. A 1.5% concentration is recommended to resolve DNA
fragments from 0.2–0.5 kb, which is only a 0.3 kb range.

Question: 6
Correct Answer: C
Explanation:
(C) Based on the information in Table 3.5, DNA fragments of 0.5–0.7 kb are relatively
small. Table 3.4 indicates that a high agarose concentration provides a sharper
resolution of small DNA fragments, and low voltage may cause fragments of less than 1
kb to diffuse in the gel. This means that a high agarose concentration (1.2%) and a high
voltage (5 V/cm) would provide the best results for DNA fragments of this size. Table
3.5 indicates that the highest agarose concentration (1.5%) is only appropriate for
fragments smaller than 0.5 kb.

Question: 7
Correct Answer: A
Explanation:
(A) According to the passage, large DNA fragments travel more slowly than small
fragments. This means the largest DNA fragments will appear closest to the cathode,
because they will have traveled the least distance from the wells.

Question: 8
Correct Answer: B
Explanation:
(B) The passage indicates that Allele 2 is larger than both Alleles 1 and 3. Therefore,
Allele 2 will travel through the agarose gel more slowly than the other two.

Question: 9
Correct Answer: D
Explanation:
(D) According to the passage, a single band that appears darker is an indicator of two
copies of the same allele. Individuals E and K both appear to have two copies of Allele
2, while N and O appear to have two copies of Allele 1.
Made by: Shahd A.Gaber

Question: 10
Correct Answer: A
Explanation:
(A) Based on information in the passage, Allele 2 is the largest of the three alleles.
Therefore, Allele 2 is indicated by the band that has traveled the least distance down
the gel. The size of this band is approximately 3.0 kb.

Question: 11
Correct Answer: B
Explanation:
(B) The combination of Alleles 1 and 2 occurs most frequently in Figure 3.3, appearing
in a total of six lanes (Lanes A, B, D, I, L, and M).

Question: 12
Correct Answer: D
Explanation:
(D) In Figure 3.3, all three alleles appear to be present in the sample in Lane J. The
most reasonable explanation is that this lane contains DNA from more than one
individual, since each individual can carry only two copies of one gene.

Question: 13
Correct Answer: C
Explanation:
(C) In Figure 3.3, all allele combinations (Alleles 1 and 1, 1 and 2, 1 and 3, etc.) are
represented in at least one lane except the combination of two copies of Allele 3.

Question: 14
Correct Answer: C
Explanation:
(C) The three alleles in Figure 3.3 appear to be approximately 2–5 kb in size. The most
appropriate agarose concentration for this size range is 1.0%. A higher concentration of
agarose is more appropriate for DNA fragments that are smaller than these, and a lower
concentration is more appropriate for larger DNA fragments.

Question: 15
Correct Answer: D
Explanation:
(D) According to Table 3.4, low agarose concentration and low voltage both cause
longer run times (possibly over multiple days). Therefore, DNA fragments should
Made by: Shahd A.Gaber

migrate most slowly when agarose concentration and voltage are both at their minimum.
This corresponds to an agarose concentration of 0.5% and a voltage of 0.25 V/cm.

(Test 32)
Question: 1
Correct Answer: B
Explanation:
(B) A stimulus is a change in the environment, such as the sounding of a tone or the
appearance of an image. A response is the individual's reaction to the stimulus. The
experiment descriptions identify the response time as the time between the sounds of a
tone or the appearance of an image (stimulus) and the student pressing the spacebar
(response).

Question: 2
Correct Answer: A
Explanation:
(A) In Experiment 1, the student pressed the spacebar in response to the sounding of a
tone, so the sounding of the tone was the stimulus.

Question: 3
Correct Answer: D
Explanation:
(D) Both Experiments 2 and 3 tested response time to auditory and visual stimuli. In
Experiment 2, three successive trials were performed using an auditory stimulus, and
then three were performed using a visual stimulus. In Experiment 3, the type of stimulus
alternated with each new trial. All other conditions were held constant between the two
experiments.

Question: 4
Correct Answer: D
Explanation:
(D) According to Table 3.7, the response time to an auditory stimulus is consistently
lower (faster) than the response time to a visual stimulus. A faster response time
indicates that auditory processing occurs faster than visual processing, suggesting that
the sense of hearing is more acute.
Made by: Shahd A.Gaber

Question: 5
Correct Answer: D
Explanation:
(D) All three experiments used a stimulus duration of 400 ms for at least some trials. In
Experiment 1, trials 4 through 6 used a 400 ms tone length. A 400 ms stimulus duration
was used on all trials during Experiments 2 and 3.

Question: 6
Correct Answer: B
Explanation:
(B) The fastest reaction time was 142 ms. This occurred during Trial 5 of Experiment 1
as well as Trials 2 and 3 of Experiment 2. All three of these trials recorded response
times to an auditory stimulus lasting a duration of 400 ms.

Question: 7
Correct Answer: A
Explanation:
(A) In Table 3.6, Trials 1 through 3 were conducted using a 200 ms tone length and
Trials 4 through 6 used a 400 ms tone length. The average response times were
approximately 10 ms faster for the trials using the longer tone.

Question: 8
Correct Answer: C
Explanation:
(B) In Tables 3.7 and 3.8, the response times for the auditory stimulus were
consistently more than 40 ms faster than for the visual stimulus. It can be inferred
that a faster response time is the result of a signal reaching the brain faster.

Question: 9
Correct Answer: A
Explanation:
(A) The average response times for auditory and visual stimuli both increased (got
slower) as Experiment 3 progressed. The average response times in Table 3.8 are also
higher than those for both stimuli in Table 3.7.
Made by: Shahd A.Gaber

Question: 10
Correct Answer: A
Explanation:
(A) The response times recorded for auditory stimuli across all three experiments range
from 142–158 ms. It is therefore reasonable to infer that the typical auditory response
time range is 140–160 ms.

Question: 11
Correct Answer: C
Explanation:
(C) According to the description of Experiment 2, 6 total trials were conducted, and each
trial consisted of 10 stimulus-response cycles. This means that a total of 60 responses
to a stimulus were recorded during Experiment 2.

Question: 12
Correct Answer: C
Explanation:
(C) Both auditory and visual response times increased over the course of Experiment 3.
Therefore, the graph for the data in Table 3.8 should consist of two lines, each with a
small positive slope. Response time to the visual stimulus was consistently higher than
to the auditory stimulus, so the visual line should be above the auditory line.

Question: 13
Correct Answer: B
Explanation:
(B) The student's new experimental design should be the same as in Experiment 1, but
using a visual stimulus (image) instead of an auditory one (tone). The student can then
test the difference in response time to an image shown for 200 ms versus an image
shown for 400 ms.

Question: 14
Correct Answer: D
Explanation:
(D) The lowest auditory response time occurred during Trial 1 of Experiment 1. The
actual response time was 158 ms.
Made by: Shahd A.Gaber

Question: 15
Correct Answer: C
Explanation:
(C) In Tables 3.6 and 3.7, the response time for a particular stimulus decreased (got
faster) over subsequent trials. In Table 3.8, the response time for a particular stimulus
increased (got slower) as the type of stimulus alternated with each trial. This suggests
that repetitive exposure to the same stimulus improves an individual's reaction time.

(Test 35)
Question: 1
Correct Answer: A
Explanation:
(A) A closed ecosystem is characterized by a lack of migration into and out of it. There
are several possible causes, one of which is geographic isolation. The passage states
that Shebay Park consists of a group of isolated islands. Since the islands are
separated from the mainland, migration to and from this ecosystem is rare.

Question: 2
Correct Answer: B
Explanation:
(B) In Figure 4.5, the peccary population is shown to feed on four different types of
plants. A consumer that feeds exclusively on plant matter is termed a herbivore.

Question: 3
Correct Answer: C
Explanation:
(C) In Figure 4.6, the peak in the jaguar population occurred around 1990. In this year,
the population consisted of 50 individuals.

Question: 4
Correct Answer: A
Explanation:
(A) In Figure 4.6, the smallest value for the peccary population was 500 individuals.
This occurred in roughly 2006.
Made by: Shahd A.Gaber

Question: 5
Correct Answer: A
Explanation:
(A) According to Figure 4.6, the peccary population experienced many consecutive
years of positive population growth beginning around 1990. Immediately preceding this
growth period, the jaguar population was devastated by exposure to feline leukemia
(1989). It can be inferred that the sharp decline in the jaguar population caused a
decrease in predatory pressure on the peccary population, thereby allowing the latter
population to increase.

Question: 6
Correct Answer: D
Explanation:
(D) Consumers in an ecosystem are identified according to how far removed they are
from the ecosystem's producers (plants). Primary consumers are herbivores that feed
only on plants. Secondary consumers feed on primary consumers. According to the
food web in Figure 4.5, two secondary consumers exist in the ecosystem—the fox and
the jaguar.

Question: 7
Correct Answer: D
Explanation:
(D) The food web in Figure 4.5 shows that nutria and duck share common food sources
and a common predator with the peccary. These similarities indicate that the three
populations occupy similar niches within the ecosystem.

Question: 8
Correct Answer: B
Explanation:
(B) In Figure 4.6, the sharp declines in the jaguar (1990) and peccary (2004)
populations coincide with the occurrence of rare environmental events. The passage
indicates that the 1990 decline in the jaguar population can be attributed to the
introduction of feline leukemia in 1989, and the 2004 decline in the peccary population
can be attributed to severe winter conditions and a tick outbreak. Each of these events
reduced the immediate ability of individuals to survive, resulting in a population size that
was drastically reduced in a matter of two years.
Made by: Shahd A.Gaber

Question: 9
Correct Answer: A
Explanation:
(A) The passage states that an outbreak of ticks occurred in 2004. This coincides with
the severe reduction in peccary population size beginning that year, suggesting that the
parasite outbreak contributed to the decrease in the population.

Question: 10
Correct Answer: D
Explanation:
(D) Several factors influencing the peccary population size are discussed in the
passage, including interactions with other species and environmental conditions.

Question: 11
Correct Answer: B
Explanation:
(B) While most species in the food web (Figure 4.5) are directly linked to the peccary,
the squirrel shares no direct connection with that population. Squirrels and peccaries
have no common predators or food sources; therefore the squirrel should be less
affected by a change in the peccary population than other species in the food web.

(Test 36)
Question: 1
Correct Answer: B
Explanation:
(B) An invasive species is both non-native to an ecosystem and harmful in some way.
Non-native species that move into a new ecosystem will disrupt that ecosystem by
competing with native species for resources and introducing new feeding relationships
into the food web.

Question: 2
Correct Answer: C
Explanation:
(C) The passage indicates that water hyacinths are able to withstand fluctuations in pH,
but it does not indicate that these plants induce changes in pH. Water hyacinths upset
freshwater ecosystems in a number of ways, but not by altering pH.
Made by: Shahd A.Gaber

Question: 3
Correct Answer: B
Explanation:
(B) In Table 5.1, the final weed combination studied was the combination of all four
weed species. This same combination is not present in Table 5.2.

Question: 4
Correct Answer: A
Explanation:
(A) Plant density is identified in the description of Study 1 as the total number of water
hyacinth plants within a sample area. A sample area was defined as 1 square meter.

Question: 5
Correct Answer: D
Explanation:
(D) The sample areas in which water hyacinth was found growing alone provide the
control group for Study 1. These sample areas indicated water hyacinth growth when
there was no competition from other weed species.

Question: 6
Correct Answer: C
Explanation:
(C) In Study 1, Justicia sp. had a slightly more negative impact on water hyacinth fresh
weight than did V. cupsidata. V. cupsidata exhibited the most negative effect on all other
growth parameters in Study 1.

Question: 7
Correct Answer: C
Explanation:
(C) In Table 5.1, the weed combination of E. crassipes and Commelina sp. exhibits the
least difference from the control group (E. crassipes alone) out of all possible
combinations. This means that Commelina sp. exerts the least competitive pressure on
water hyacinth.

Question: 8
Correct Answer: D
Explanation:
(D) In Table 5.2, water hyacinths grown alone were shown to have an average height of
8.69 cm. When grown with Commelina sp. and Justicia sp. in a greenhouse, the
average water hyacinth height was shown to increase to 8.8 cm and 8.88 cm,
Made by: Shahd A.Gaber

respectively. The increase in growth suggests a positive effect of these competitor


species on water hyacinth height.

Question: 9
Correct Answer: A
Explanation:
(A) The number of leaves per plant for a water hyacinth grown alone was higher in
Table 5.2 than in Table 5.1. The values for fresh weight and plant height were both
much lower in Table 5.2 than in Table 5.1.

Question: 10
Correct Answer: A
Explanation:
(A) In the description of Study 1, total biomass was described as being calculated by
multiplying plant density by fresh weight. Since plant density is a component of total
biomass and was not recorded in Study 2, total biomass could not be calculated for this
study.

Question: 11
Correct Answer: A
Explanation:
(A) In both studies, V. cupsidata was shown to decrease water hyacinth growth by all
parameters, except for fresh weight in Study 1. These data support the claim that V.
cupsidata has the most negative impact on water hyacinth growth.

Question: 12
Correct Answer: B
Explanation:
(B) In Table 5.2, all three competitor weeds were shown to reduce water hyacinth fresh
weight, thus leading to lighter plants than when water hyacinth grew alone. For both
plant height and leaves per plant, at least one competitor species was shown to
increase water hyacinth growth.

Question: 13
Correct Answer: C
Explanation:
(C) In Study 2, scientists removed young weed plants from the Kagera River and grew them in a
greenhouse. Within the greenhouse, the researchers had more control over the environmental
conditions to which the plants were exposed, thus controlling any environmental factors (other
than competitor species) that may influence water hyacinth growth.
Made by: Shahd A.Gaber

Question: 14
Correct Answer: D
Explanation:
(D) In Table 5.1, the average fresh weight of water hyacinth growing in the presence of
all three competitor species was 342 g. This value is approximately equal to the mean
(average) water hyacinth fresh weight of 368 g when growing in the presence of each
competitor species individually.

Question: 15
Correct Answer: D
Explanation:
(D) Throughout both studies, V. cupsidata consistently showed the most negative effect
on water hyacinth growth. This is true for all growth parameters except one (fresh
weight in Study 1). Commelina sp. and Justicia sp. both showed mixed effects on water
hyacinth growth, depending on the growth parameter observed and the study
environment. Therefore, increasing the presence of V. cupsidata alone can be predicted
to significantly reduce the water hyacinth population.

(Test 38)
Question: 1
Correct Answer: A
Explanation:
(A) According to Table 5.5, all organisms in the phylogenetic tree are members of the
class Mammalia. This means that all of the organisms are mammals.

Question: 2
Correct Answer: C
Explanation:
(C) Panthera pardus is the scientific name for leopard. Table 5.5 identifies the leopard
as belonging to the family Felidae.

Question: 3
Correct Answer: B
Explanation:
(B) All organisms listed in Table 5.5 share a common kingdom, phylum, class, and
order. The organisms diverge into different taxa beginning with the family level.
Made by: Shahd A.Gaber

Question: 4
Correct Answer: D
Explanation:
(D) Table 5.5 lists Canis latrans as the genus and species of the coyote.

Question: 5
Correct Answer: C
Explanation:
(C) In a phylogenetic tree, a species is most closely related to the species with which it
shares the most recent common ancestor, or node. According to Figure 5.6, the
European otter (Lutra lutra) shares the most recent common ancestor with Taxidea
taxus.

Question: 6
Correct Answer: B
Explanation:
(B) In a phylogenetic tree, the number of common ancestors shared by two species is
indicated by the number of nodes shared by those species. In Figure 5.6, Panthera
pardus diverges from the other four species at the very first node, indicating that it
shares only one common ancestor with the other four species.

Question: 7
Correct Answer: D
Explanation:
(D) The passage states that the length of the horizontal lines on a phylogenetic tree
indicates the relative divergence time between species. Canis latrans and Canis
lupus are connected to their most recent common ancestor by the shortest lines,
indicating that these two species diverged most recently.

Question: 8
Correct Answer: A
Explanation:
(A) The passage identifies an extant species as one that is currently living. Taxidea
taxus, listed along the right side of the phylogenetic tree in Figure 5.6, is currently living.
Made by: Shahd A.Gaber

Question: 9
Correct Answer: B
Explanation:
(B) The seven-level classification system used in Table 5.5 classifies organisms using a
hierarchical system that goes from broadest grouping (kingdom) to most specific
(species). As shown in Table 5.5, organisms that share a certain taxonomic level also
share the same higher taxonomic levels, but they may diverge in lower levels. This
means that species belonging to the same order must also belong to the same phylum
and kingdom, but they may belong to a different order.

Question: 10
Correct Answer: C
Explanation:
(C) The lynx belongs to the family Felidae. Panthera pardus also belongs to this family.
Based on the phylogenetic tree in Figure 5.6, the lynx shares the most recent common
ancestor with Panthera pardus.

Question: 11
Correct Answer: D
Explanation:
(D) The lynx belongs to a different family than the gray wolf, but the same taxa for order
and above. This means that the lynx and gray wolf share four common taxonomic
levels.

Question: 12
Correct Answer: B
Explanation:
(B) The five species in Table 5.5 belong to three different families, but they all belong to
the same order (Carnivora). This means the most recent common ancestor shared by
all five species was also a member of the Carnivora taxon.

Question: 13
Correct Answer: C
Explanation:
(C) A clade must include all extant taxa that have descended from a particular ancestral
taxon. In Figure 5.6, Canis latrans, Canis lupus, and Lutra lutra do not constitute a
complete clade because their most recent common ancestor is also the ancestor
of Taxidea taxus.
Made by: Shahd A.Gaber

Question: 14
Correct Answer: D
Explanation:
(D) According to Table 5.5, members of the family Mustelidae are also members of the
order Carnivora. This is the same order to which the coyote belongs, since all species in
Table 5.5 belong to this order. There is not enough information to determine to which
member of Mustelidae in Table 5.5 the wolverine is more closely related.

Question: 15
Correct Answer: A
Explanation:
(A) According to Figure 5.6, the member of the family Felidae (Panthera
pardus) diverged from the other four species first. The members of Mustelidae and
Canidae are then shown to diverge from each other next. This means the Canidae
taxon diverged from the Mustelidae taxon more recently than from the Felidae taxon.

(Test 39)
Question: 1
Correct Answer: C
Explanation:
(C) The code for building a protein is stored in the cell's DNA. As with any protein, the
first step in building a tropomyosin protein is transcribing the α-TM gene into mRNA.
After RNA splicing, this mRNA transcript can be used to build the protein.

Question: 2
Correct Answer: A
Explanation:
(A) According to the passage, different mRNA transcripts can be produced from the
same α-TM gene. Figure 6.1 shows seven different α-TM mRNA transcripts that
scientists discovered were produced from the same α-TM gene.

Question: 3
Correct Answer: A
Explanation:
(A) The maximum number of exons present in an mRNA transcript is 10. Both types of
striated muscle exhibit mRNA transcripts with 10 exons.
Made by: Shahd A.Gaber

Question: 4
Correct Answer: D
Explanation:
(D) Exon 4 is present in all seven transcripts in Figure 6.1. This suggests that Exon 4 is
a constitutive exon necessary for all tropomyosin proteins.

Question: 5
Correct Answer: B
Explanation:
(B) The mRNA transcript found in the brain contains six constitutive exons and one
alternatively spliced exon. Exon 7 is present in the brain but not in all tissue types.

Question: 6
Correct Answer: C
Explanation:
(C) Exons 2 and 3 do not appear together in any of the seven mRNA transcripts in
Figure 6.1. Exons 11 and 12 also do not appear together.

Question: 7
Correct Answer: D
Explanation:
(D) Since two samples of striated muscle are shown in Figure 6.1, it can be assumed
that one sample is skeletal muscle and the other is cardiac muscle. Examining both
mRNA transcripts shows that the sample labeled Striated muscle ends with Exon 11,
while the sample labeled Striated muscle′ ends with Exon 12. No other differences exist
between the two types.

Question: 8
Correct Answer: B
Explanation:
(B) According to Figure 6.2, the α-TM gene contains 12 total exons. Though the number
of exons in the mRNA transcripts varies, the number of exons in the actual gene does
not change.

Question: 9
Correct Answer: B
Explanation:
(B) According to Table 6.1, Exon 4 codes for amino acids 81–125. This is a total of 44
amino acids, which is the greatest number of amino acids coded by any single exon in
the table.
Made by: Shahd A.Gaber

Question: 10
Correct Answer: D
Explanation:
(D) According to Table 6.1, either Exon 10 or 12 codes for amino acids 258–284. The
brain mRNA transcript contains neither Exon 10 or 12.

Question: 11
Correct Answer: A
Explanation:
(A) Table 6.1 shows that Exons 10 and 12 both code for amino acids 258–284. Though
all other transcripts only contain one of the two exons, striated muscle' contains both.
This means that the striated muscle' mRNA transcript contains two copies of the code
for amino acids 258–284.

Question: 12
Correct Answer: B
Explanation:
(B) Based on Figure 6.1, the difference between a myoblast and a smooth muscle
transcript involves Exons 2 and 3. Exon 3 is present in the myoblast transcript but is
absent from the smooth muscle transcript, where it is replaced by Exon 2.

Question: 13
Correct Answer: C
Explanation:
(C) According to Table 6.1, Exon 11 codes for no amino acids. This suggests that Exon
11 must contain an untranslated region instead of an amino acid code.

Question: 14
Correct Answer: A
Explanation:
(A) In Figure 6.1, the first four mRNA transcripts are from different types of muscle (or
muscle precursor) cells. The last three transcripts are from different types of nonmuscle
cells. All four of the muscle cell transcripts contain either Exon 2 or Exon 3. None of the
nonmuscle transcripts contain either of these exons. Therefore, it can be inferred that
the extra function of tropomyosins in muscle cells could be related to the sequence of
amino acids (amino acids 39–80) coded by Exons 2 and 3.
Made by: Shahd A.Gaber

Question: 15
Correct Answer: D
Explanation:
(D) According to Figure 6.1, the hepatoma is the only tissue type from which Exon 7 is
missing. This implies that a loss of Exon 7 may be correlated to tumor formation.

(Test 40)
Question: 1
Correct Answer: C
Explanation:
(C) The passage identifies neonicotinoids as a class of pesticides thought to be harmful
to honeybees.

Question: 2
Correct Answer: B
Explanation:
(B) Scientist 1 states that neonicotinoids have been shown to increase honeybees'
susceptibility to disease and parasites. Though no direct link between neonicotinoids
and honeybee population loss has been found, Scientist 1 believes neonicotinoids to be
an indirect factor.

Question: 3
Correct Answer: A
Explanation:
(A) Scientist 1 lists almonds, apples, onions, and carrots as crops that rely almost
exclusively on honeybees for pollination. Cherries are not mentioned.

Question: 4
Correct Answer: B
Explanation:
(B) Scientist 1 states that any measures with the potential to improve honeybee health,
such as a neonicotinoid ban, should be attempted. Scientist 2 states that a
neonicotinoid ban should not be attempted because neonicotinoids are not among the
greatest threats to honeybee health, and efforts should instead be focused on the
strongest known threats.
Made by: Shahd A.Gaber

Question: 5
Correct Answer: A
Explanation:
(A) According to Scientist 1, reproductive rates of honeybee-dependent crops vary
directly with the availability of honeybees. This would also apply to honeybee-
dependent producers in the natural ecosystem. Figure 6.3 demonstrates a direct
relationship between honeybee population size and producer reproductive rate.
As honeybee population size increases, so does the producer reproductive rate.

Question: 6
Correct Answer: A
Explanation:
(A) Scientist 2 states that varroa mites and disease are the greatest known threats to
honeybee health and suggests that efforts to improve that health should focus on these
threats.

Question: 7
Correct Answer: D
Explanation:
(D) Scientist 2 identifies water that is scarce or of poor quality as an environmental
stressor to honeybees. Scientist 1 does not mention water quality as a factor affecting
honeybee health.

Question: 8
Correct Answer: C
Explanation:
(C) Scientist 1 states that honeybees are important to the overall ecosystem because
plants at the base of the food web depend on pollination by honeybees. This indicates
that Scientist 1 believes transferring pollen between plants is honeybees' most
important contribution to natural ecosystems.

Question: 9
Correct Answer: D
Explanation:
(D) Scientist 2 indicates that greater exposure to neonicotinoids does correlate slightly
to a decrease in honeybee population size. Figure 6.10 demonstrates a weak negative
correlation between pesticide exposure and honeybee population size. Though the data
points do not form a tight line, a negative average slope is identifiable.
Made by: Shahd A.Gaber

Question: 10
Correct Answer: C
Explanation:
(C) The passage states that the European Union recently instituted a two-year ban on
neonicotinoids in an effort to improve the health of European honeybee populations. A
doubling of European honeybee populations over the next five years would indicate that
the neonicotinoid ban was effective in improving honeybee health. This supports the
opinion of Scientist 1, who believes a neonicotinoid ban would be effective in the United
States.

Question: 11
Correct Answer: B
Explanation:
(B) Both scientists discussed disease and parasites (varroa mites) as contributing to the
decline of honeybee populations. Scientist 2 asserts that disease and parasites are the
greatest threats to honeybee health. Scientist 1 asserts that any measures available to
improve honeybee health should be taken. Therefore, it can be inferred that both
scientists would support efforts to improve disease and parasite prevention.

Question: 12
Correct Answer: A
Explanation:
(A) Scientist 1 states that 23% of crops in the United States rely on honeybee
pollination. This percentage accounts for approximately? of the agriculture industry.

Question: 13
Correct Answer: C
Explanation:
(C) According to Scientist 2, a ban on neonicotinoids would not be effective in
increasing honeybee populations because neonicotinoids are only weakly correlated to
honeybee population declines. This weak correlation would lead honeybee populations
to continue to decline during a neonicotinoid ban but at a slightly slower rate.

Question: 14
Correct Answer: D
Explanation:
(D) Scientist 2 identifies disease and varroa mites as the greatest threats to honeybee
health. So it can be inferred that these biotic, or living, factors have the strongest effect
on honeybee populations.
Made by: Shahd A.Gaber

(Test 41)
Question: 1
Correct Answer: A
Explanation:
(A) Most reef-building corals have a mutually beneficial symbiotic relationship with a
microscopic unicellular alga called zooxanthellae that lives within the cells of the coral's
stomach. The coral provides the algae with a protected environment, and the
compounds necessary for photosynthesis.

Question: 2
Correct Answer: A
Explanation:
(A) As the concentration of CO2 in the water increases, the pH decreases and the
balance between bicarbonate and carbonate shifts increasingly toward bicarbonate as
the ocean attempts to buffer the drop in pH by combining H+ with CO32– to produce
HCO3–. As the carbonate ion concentration decreases, it becomes more difficult for the
corals to extract the CO32– from the seawater to build their skeletons. It is presently
unknown how species vary in their ability to cope with the decrease in carbonate ion
concentration in a process known as acclimation.

Question: 3
Correct Answer: C
Explanation:
(C) Ocean acidification, like global warming, is a predictable response to those human
activities that increase the atmospheric concentration of carbon dioxide. The magnitude
and rate of ocean acidification can be predicted with more confidence than the rise in
temperature due to global warming, as they are less dependent on climate-system
feedbacks.

Question: 4
Correct Answer: B
Explanation:
(B) Seasonal changes such as those in temperature and bioproductivity, including
variations in photosynthesis and respiration, contribute to fluctuations in ocean pH.
Coastal waters are more likely to be affected by the terrestrial system, such as runoff
from rivers, leading to wider variations in ocean pH in these areas.
Made by: Shahd A.Gaber

Question: 5
Correct Answer: C
Explanation:
(C) Crustaceans live below the saturation point and show a significant increase in
calcification, unlike gastropods, corals, and calcareous algae.

Question: 6
Correct Answer: A
Explanation:
(A) The aragonite form of calcium carbonate is more soluble than calcite because
the aragonite saturation horizon is always nearer the surface of the oceans than
the calcite saturation horizon. Therefore, calcifying organisms that produce the
calcite form of calcium carbonate (coccolithophores and foraminifera) may be
less vulnerable to changes in ocean acidity than those that construct aragonite
structures (corals and pteropods).

Question: 7
Correct Answer: B
Explanation:
(B) Figure 6.12 shows that bivalve fertility is also negatively impacted by acidification,
with slowed development and decreased fertilization and larval survival.

Question: 8
Correct Answer: A
Explanation:
(A) As noted in Figure 6.12, the decline in coral calcification is noted as significant with
an asterisk.

Question: 9
Correct Answer: C
Explanation:
(C) Calcareous algae is most affected, showing a significant decline in growth and
photosynthesis, as well as a decrease in reproduction.

Question: 10
Correct Answer: C
Explanation:
(C) Most reef-building corals have a mutually beneficial, symbiotic relationship with the
microscopic unicellular algae called zooxanthellae. The coral provides the algae with a
protected environment and the compounds necessary for photosynthesis.
Made by: Shahd A.Gaber

Question: 11
Correct Answer: D
Explanation:
(D) The outer epidermis provides a layer of protection over the coral animal.

Question: 12
Correct Answer: D
Explanation:
(D) Coral eggs combining with sperm in sexual reproduction produces the most genetic
diversity.

Question: 13
Correct Answer: A
Explanation:
(A) Coral bleaching occurs when the corals lose their color due to stress-induced
expulsion of the symbiotic unicellular algae.

Question: 14
Correct Answer: A
Explanation:
(A) The best design would be to use two tanks filled with seawater and corals, adding
carbon dioxide bubbles to only one tank. The tank with added carbon dioxide is the
variable, while the other serves as the experimental control.

Question: 15
Correct Answer: B
Explanation:
(B) Any changes in the biological processes in the surface ocean waters also affect the
deeper water. This is because organisms and habitats living at the lower levels of the
oceans—far from sunlight—rely mainly on the products created by life in the surface
waters. On a longer timescale, these organisms may also be vulnerable to acidification
and changes in ocean chemistry as higher levels of carbon dioxide mix throughout the
oceans.
Made by: Shahd A.Gaber

(Test 43)
Question: 1
Correct Answer: B
Explanation:
(B) This piece of evidence involves the reduction of predators, which is Scientist A's
primary view on the issue. All other choices represent evidence that would be presented
by Scientist B regarding the lack of competition for resources from other herbivores.

Question: 2
Correct Answer: C
Explanation:
(C) Both scientists agree on the mechanism of the demise of the deer population; their
opinions differ on how the population grew and, on the data, presented.

Question: 3
Correct Answer: C
Explanation:
(C) This statement focuses only on the predator-prey relationship, and Scientist B tends
to downplay the effects of predator reduction in the description of the Kaibab Plateau.

Question: 4
Correct Answer: A
Explanation:
(A) Both scientists agree on the negative implications of interfering in ecosystems.

Question: 5
Correct Answer: D
Explanation:
(D) Scientist A focuses on food chain disruption, especially in regard to removing
predators.

Question: 6
Correct Answer: D
Explanation:
(D) The lesson of this situation, according to Scientist A, is that predators are a
necessary part of the ecosystem and removing them can have catastrophic results.
Made by: Shahd A.Gaber

Question: 7
Correct Answer: B
Explanation:
(B) The scientists disagree on how the population increased (lack of predation versus
lack of competition for resources), but the reason the population decreased
catastrophically is clear—disease and starvation from overpopulation.

Question: 8
Correct Answer: D
Explanation:
(D) Scientist B believes that the reduction in competition for grasses caused the
increase in the population.

Question: 9
Correct Answer: B
Explanation:
(B) While the other statements could potentially be true, this cautionary statement
shows a link between the practice suggested for increasing the duck population
and what happened on the Kaibab. Removing the midsized mammals (raccoons,
foxes, and so forth) in this ecosystem could have a similar ripple effect that
removing the large predators (such as coyotes) had on the Kaibab Plateau.

Question: 10
Correct Answer: B
Explanation:
(B) Although the two scientists differ in their reactions to the environmental mechanisms
that led to the problem, they agree that a lesson should be learned from the Kaibab
Plateau situation about human intervention in natural ecosystems.
Made by: Shahd A.Gaber

(Test 49)
Question: 1
Correct Answer: B
Explanation:
(B) The target for the North Lake area is 25 μg/L. In all years of the study, the values of
phosphorus measured in North Lake have exceeded that level.

Question: 2
Correct Answer: B
Explanation:
(B) According to Table 8.6, a phosphorus level between 12 μg/L and 24 μg/L would be
classified as mesotrophic. The target for Bay Area 2 is 14 μg/L.

Question: 3
Correct Answer: C
Explanation:
(C) All of the areas except for South Lake were above their target phosphorus level in
2011. South Lake was at 48 μg/L with a goal of 54 μg/L or below.

Question: 4
Correct Answer: D
Explanation:
(D) South Lake is the only area that shows a downward trend in phosphorus levels
starting in 2005.

Question: 5
Correct Answer: C
Explanation:
(C) In 2002, the South Lake phosphorus level was 38 μg/L. This falls in the category of
eutrophic.

Question: 6
Correct Answer: D
Explanation:
(D) Bay Area 2 fell in the range of 12 μg/L to 24 μg/L for 19 years of the study.
Made by: Shahd A.Gaber

Question: 7
Correct Answer: D
Explanation:
(D) Bay Area 1 fell in the range of 12 μg/L to 24 μg/L during 2011.

Question: 8
Correct Answer: A
Explanation:
(A) By subtracting the approximate value of the shortest bar from that of the highest bar
in Figure 8.10, a range can be determined (61 – 21 = 32 μg/L).

Question: 9
Correct Answer: A
Explanation:
(A) Bay Area 1 had a range of only 5 μg/L (13 – 8 = 5 μg/L). It is important to pay
attention to the scale in each figure, as each has a different unit scale on the y-axis. The
ranges for each lake area can be found by subtracting the approximate value of the
shortest bar from that of the highest bar in each figure.

Question: 10
Correct Answer: B
Explanation:
(B) According to the passage, the value must be at or below the target value of
phosphorus to surpass the standard. By counting the bars at or below 10 μg/L in Figure
8.7, one can determine that the standard was met or surpassed for 10 years in Bay
Area 1.

Question: 11
Correct Answer: B
Explanation:
(B) In 1993, Bay Area 1 had an approximate phosphorus concentration of 9 μg/L and
North Lake had a concentration of 50 μg/L (50 – 9 = 41 μg/L).

Question: 12
Correct Answer: A
Explanation:
(A) In 2005, Bay Area 1 had the most similar phosphorus concentration to that of the
neighboring lake.
Made by: Shahd A.Gaber

Question: 13
Correct Answer: D
Explanation:
(D) To be classified as hypereutrophic, the phosphorus level of a lake needs to be
higher than 96 μμg/L. None of the lake areas exceeded that level during the 21-year
study.

Question: 14
Correct Answer: C
Explanation:
(C) The North and South Lake areas most frequently fell into the range of 24 μg/L to 96
μg/L.

Question: 15
Correct Answer: B
Explanation:
(B) According to the passage, lakes classified as eutrophic have low levels of dissolved
oxygen. In 2010, North Lake was the most eutrophic of all the areas studied and would
therefore be expected to have the lowest dissolved oxygen level.

(Test 59)
Question: 1
Correct Answer: C
Explanation:
(C) Chlorophyll a and b have the lowest absorption in the 525 to 625 nm portion of the
spectrum, which corresponds to the greatest reflection. Even without a knowledge of
light principles, this can be determined through the process of elimination, since the
other answers do not match the data in Figure 11.1.

Question: 2
Correct Answer: B
Explanation:
(B) The blue portion of the visible spectrum is between 450 and 495 nm.

Question: 3
Correct Answer: B
Explanation:
(B) A wavelength of 440 nm represents considerable absorption for all three pigments.
Made by: Shahd A.Gaber

Question: 4
Correct Answer: A
Explanation:
(A) From Figure 11.1, it can be determined that red light has wavelengths greater than
620 nm. Chlorophyll a has the highest peak in the range of wavelengths greater than
620 nm.

Question: 5
Correct Answer: D
Explanation:
(D) Carotenoids absorb 50% of visible light at 450 nm, and chlorophyll b absorbs 80%,
which means the carotenoids have? (62.5%, or approximately 60%) of the absorbing
power at that wavelength.

Question: 6
Correct Answer: A
Explanation:
(A) The pigment with the highest absorption at 425 nm is chlorophyll a.

Question: 7
Correct Answer: B
Explanation:
(B) The highest peak for chlorophyll b in Figure 11.1 is at 450 nm.

Question: 8
Correct Answer: C
Explanation:
(C) Absorption and reflection are opposites, but one would not have to be familiar with
the term or process of reflection/principles of color to determine that green light is
reflected by chlorophyll a and b, making plants appear green. Figure 11.1 shows that
(A) and (B) are false. (D) is the opposite of (C) and can be eliminated by looking at
Figure 11.1, which shows higher absorption in the red portion of the spectrum than in
the green.

Question: 9
Correct Answer: C
Explanation:
(C) The peaks of this pigment are the opposite of those of chlorophyll a and b, as it
absorbs the most light in the green portion of the visible spectrum where the absorption
of chlorophyll is the lowest.
Made by: Shahd A.Gaber

(Test 62)
Question: 1
Correct Answer: B
Explanation:
(B) The data for Group 1 show S. aureus growing most rapidly and to the highest
turbidity at 37°C.

Question: 2
Correct Answer: C
Explanation:
(C) At 600 minutes, the data reached their maximum value. Visualizing a graph of this
data, one would see it level off at this point, corresponding to the stationary phase of
growth.

Question: 3
Correct Answer: D
Explanation:
(D) pH is one of the variables that is controlled for Group 1 and is the variable being
tested by Group 2.

Question: 4
Correct Answer: B
Explanation:
(B) Twenty-nine minutes most closely approximates the generation time. One can
choose any two pieces of data up to 100 minutes where the absorbance value doubles
(for example, 0.047 and 0.099) and determine the amount of time that has passed by
subtracting the time readings (68 and 97 minutes, respectively). Even if other data
points are selected (such as 0.035 and 0.070) or one chooses another method of
approximation, (C) is the only answer that is reasonable.

Question: 5
Correct Answer: B
Explanation:
(B) The temperature of incubation is the experimental variable for Group 1, not one of
the controls.
Made by: Shahd A.Gaber

Question: 6
Correct Answer: D
Explanation:
(D) S. aureus has a faster growth rate at pH 5 than at pH 7, as evidenced by the more
rapid increase in absorbance values in Table 11.5.

Question: 7
Correct Answer: B
Explanation:
(B) The information in the passage provides the distinction between acidity and
alkalinity. At this high level of alkalinity, S. aureus bacteria cannot grow, as evidenced
by the lack of increase in turbidity.

Question: 8
Correct Answer: C
Explanation:
(C) Group 1/Trial 3 and Group 2/Trial 3 were both performed under conditions of 37°C
and pH 6.

Question: 9
Correct Answer: A
Explanation:
(A) It can be concluded from the growth data that 37°C and pH 6 represent the ideal
growth conditions for S. aureus.

Question: 10
Correct Answer: A
Explanation:
(A) The graph will be a curve, as the data increases and then levels off between 10 and
24 hours.

Question: 11
Correct Answer: C
Explanation:
(C) This trial is the only one mentioned that occurred at 37°C and featured growth that
was retarded but not completely inhibited by the conditions.
Made by: Shahd A.Gaber

Question: 12
Correct Answer: A
Explanation:
(A) Time is the independent variable, as its presence determines the value of the other
variables.

Question: 13
Correct Answer: C
Explanation:
(C) Comparing generation time at a standard temperature would be the most accurate
way to determine the type of bacteria using spectrophotometry. Although (A) and (D)
are true of S. aureus, they are likely true of many other bacteria as well and lack the
specificity of a calculation such as generation time.

Question: 14
Correct Answer: B
Explanation:
(B) The population (as indicated by turbidity) remained relatively stable, which means
that birth and death must be balanced.

(Test 67-1)
Question: 1
Correct Answer: B
Explanation:
(B) In the third sentence of her first paragraph, Researcher 2 mentions ozone as one of
the air pollutants that had their concentrations lowered by a decrease in traffic, which
also led to a decrease in asthma-related emergencies.Therefore, she would expect a
decrease in ozone levels to lead to a decrease in the prevalence of asthma.

Question: 2
Correct Answer: H
Explanation:
(H) Researcher 1 thinks that asthma is caused by allergens, and mentions dogs and
cats as being among these allergens in the middle of her first paragraph. Therefore, she
would view pet owners as more likely to have asthma.
Made by: Shahd A.Gaber

Question: 3
Correct Answer: A
Explanation:
(A) In her second paragraph, Researcher 1 states that high IgE levels and asthma are
positively correlated, and that a "high" IgE level is considered to be one above 200
IU/ml. Therefore, she would regard someone with an IgE level of 400 IU/ml as being
significantly at risk for asthma.

Question: 4
Correct Answer: J
Explanation:
(J) In her first paragraph, Researcher 1 names indoor allergens as a cause of asthma,
and suggests that less exposure to outside air increased the detrimental effects of these
allergens (Houses are built more tightly and are better insulated). Therefore, she would
expect more circulation and more outside air to lessen the prevalence of asthma.

Question: 5
Correct Answer: D
Explanation:
(D) Researcher 1 blames asthma on indoor allergens, and Researcher 2 blames
asthma on air pollution. Since these are both things that people can do something
about, both researchers would agree that measures can be taken to lower a person's
risk of developing asthma.

Question: 6
Correct Answer: G
Explanation:
(G) Researcher 2 names CO as one of the air pollutants linked to high rates of asthma-
related emergencies. Therefore, she would expect CO concentration and asthma rates
to increase concurrently. Only choice G depicts a graph where CO and cases of asthma
increase together, as indicated by the line going steadily upward from left to right.

Question: 7
Correct Answer: C
Explanation:
(C) Researcher 2 regards air pollution (of which high ozone levels are a component) as
a cause of asthma. Therefore, she would expect asthma rates to have been lower 20
years ago when there was less air pollution (including lower ozone levels).
Made by: Shahd A.Gaber

(Test 68-1)
Question: 1
Correct Answer: A
Explanation:
(A) If the new vaccine introduced in 1991 had really been more effective, then rates of
pertussis would have dropped thereafter, but instead they increased (the line starts
going upward). Therefore, the data in Figure 2 do not support this claim.

Question: 2
Correct Answer: H
Explanation:
(H) According to Figure 1, the only stage in which someone would have a cough and be
contagious is the paroxysmal stage. Although Figure 1 establishes that medication can
significantly shorten the period of time that one is contagious, the fact that the person in
the question is medicated is superfluous information. (Questions on the ACT Science
frequently provide more information than is necessary to answer the question!)

Question: 3
Correct Answer: C
Explanation:
(C) Figure 1 states that the two stages in which a patient is contagious are the
catarrhal and the paroxysmal. It also shows that the maximum amount of time
the catarrhal stage can last is 2 weeks, and that the maximum amount of time the
paroxysmal stage can last is 6 weeks. Therefore, the maximum amount of time a
patient can be contagious is 8 weeks.

Question: 4
Correct Answer: G
Explanation:
(G) The highest bars in Figure 3 are the ones representing the 10–14 and 15–19 age
groups, and so Figure 3 does not support the idea that young children are the most
susceptible to pertussis.

Question: 5
Correct Answer: D
Explanation:
(D) The claim that vaccine immunity wanes after 5–10 years is consistent with the data
in Figure 3, which indicates the greatest numbers of pertussis cases are in children
ages 10–19.
Made by: Shahd A.Gaber

Question: 6
Correct Answer: H
Explanation:
(H) Figure 1 states that antibiotics make a big difference in how long pertussis
communicability lasts. Therefore, the assertion that pertussis cannot be treated with
antibiotics is the one that is NOT true.

(Test 69-1)
Question: 1
Correct Answer: G
Explanation:
(G) Table 1 establishes that a female who has normal vision but is a carrier of red-green
color blindness would have one normal X chromosome and one mutated X
chromosome. (It is also entirely possible to figure this out without Table 1, assuming you
knew that women have two X chromosomes and a bit about how heritability works.)

Question: 2
Correct Answer: D
Explanation:
(D) For a couple to be assured of producing only red-green color-blind children, both
parents would have to be red-green color blind. The genotypes in choice D represent a
red-green color-blind female and a red-green colorblind male.

Question: 3
Correct Answer: F
Explanation:
(F) The top right corner of the Punnett square in Figure 3 (Study 2) represents a male
with normal vision. All of the males represented in Figures 2 and 4 (in Studies 1 and 3)
are red-green color blind.

Question: 4
Correct Answer: B
Explanation:
(B) One cannot determine the genetic possibilities from the chart. One cannot assume
that all the genetic possibilities would happen twice or assume that there would be four
boys or girls.
Made by: Shahd A.Gaber

Question: 5
Correct Answer: H
Explanation:
(H) Study 1 establishes that all of Barbara and John's male children are red-green color
blind, while none of their female children are (though they all carry the mutated gene).
Figure 1 establishes that Barbara and John have two girls and two boys; that is, they
have an equal number of red-green color-blind children and children with normal vision,
for a ratio of 1:1 (2:2 reduced).

Question: 6
Correct Answer: B
Explanation:
(B) A red-green color-blind female would have to have had a red-green color-blind
father and a mother who was either red-green color-blind herself or a carrier. Only
choice B presents one of these options (red-green color-blind father and carrier mother).

(Test 71-1)
Question: 1
Correct Answer: B
Explanation:
(B) Nutrition or the lack thereof is an environmental factor, and does not alter an
individual's DNA, so if ill effects due to a grandparent's malnutrition are turning up in
grandchildren, then this would be an example of "soft inheritance."

Question: 2
Correct Answer: J
Explanation:
(J) The passage does not say who proposed the theory of epigenetics. It only mentions
that the discovery was very recent.

Question: 3
Correct Answer: C
Explanation:
(C) Scientist 1 states that it would be premature to acknowledge true soft inheritance in
animal species. This implies that she might be willing to concede its existence in plant
species (as many otherwise skeptical biologists indeed do). An apple tree is the only
one of the choices that is a plant.
Made by: Shahd A.Gaber

Question: 4
Correct Answer: G
Explanation:
(G) A believer in epigenetic soft inheritance would disagree with the idea than genes act
in isolation to produce traits. In other words, he or she would believe that an organism's
expressed traits are the result partly of genes and partly of other factors.

Question: 5
Correct Answer: D
Explanation:
(D) Scientist 1 dismisses apparent evidence for epigenetic theory by arguing that the
inherited acquired traits would work themselves out of the bloodline in time ("the
bloodline always reverts to the true expression of its DNA,"and the map and wind
analogy).

Question: 6
Correct Answer: J
Explanation:
(J) If histones are the modules through which genes express themselves, and Scientist
2 figures genes (i.e., "sequences of DNA") as light bulbs, then it follows that histones
would be analogous to the lamp.

Question: 7
Correct Answer: A
Explanation:
(A) Scientist 1's defense against all epigenetic arguments is to imply that all apparently
epigenetic effects are only temporary. And Scientist 2 appears to have no proof so far
that they are not. Only a conclusive study of how long the altered expression of a gene
truly lasts (which would take a pretty long time) can settle the matter for good.
Made by: Shahd A.Gaber

(Test 71-2)
Question: 8
Correct Answer: G
Explanation:
(G) This is a simple "Go Up, Hit Line, Go Over" question. In Figure 2, right above the 5-
hour mark on the x-axis (horizontal), the line representing sleep stages is at a height of
around 1 on the y-axis (vertical).

Question: 9
Correct Answer: C
Explanation:
(C) The line in Figure 1, which represents the sleep pattern of a child, hits the bottom of
the graph (Stage 0, or being awake) only once. The 0 at the left of the graph is not
included because this represents the beginning of sleep.

Question: 10
Correct Answer: F
Explanation:
(F) The subjects of the figures get increasingly older, and the line in Figure 3 hits bottom
more than the lines in the other figures do. Therefore, the conclusion that people wake
up more frequently as they age is supported by the data. Remember, each touch at the
bottom is stage 0 and stage 0 indicates awake.

Question: 11
Correct Answer: D
Explanation:
(D) The introductory paragraph states that REM sleep is associated with the final third
of a sleep cycle. Of the answer choices, only the 7-hour mark is in the final third of the
sleep cycle depicted in Figure 2, which represents a young adult (although since the
passage states that REM sleep occurs in the final third of a sleep cycle for everyone, it
wouldn't matter which figure you looked at).

Question: 12
Correct Answer: F
Explanation:
(F) The line in Figure 1, which represents the sleep pattern of a child, is at the Stage-4
mark for an extended plateau at the 2-hour mark.
Made by: Shahd A.Gaber

(Test 73-2)
Question: 6
Correct Answer: A
Explanation:
(A) As shown in Table 1, parents with blood types of O and B can only produce
offspring with O or B blood as well. Whether the mother is O or B, whether the father is
O or B, the results are the same.

Question: 7
Correct Answer: G
Explanation:
(G) As Tables 1 and 2 make clear, to produce a child with blood type AB+, at least one
parent must have a blood type with A in it (A or AB), at least one parent must have a
blood type with B in it (B or AB), and at least one parent must be Rh+. The only one of
the choices that has an A, a B, and a + in it is choice G.

Question: 8
Correct Answer: C
Explanation:
(C) Table 3 shows that the "universal donor" is type O–. Table 1 shows that a child with
type O blood could not possibly have a parent who has type AB blood
(i.e., neither parent can be AB). When both parents have B– blood, B– or O– offspring
will be produced.

Question: 9
Correct Answer: G
Explanation:
(G) Parents with blood types A and O can produce either A or O offspring (Table 1). The
Rh factor could be + OR – because both parents could carry the recessive gene for Rh–

Question: 10
Correct Answer: B
Explanation:
(B) In the opening paragraph, the passage clearly states, "The genes ensure that only
the blood cells of the proper blood type remain in the body"—that is, they control the
types of cells in the blood.
Made by: Shahd A.Gaber

(Test 74-1)
Question: 1
Correct Answer: F
Explanation:
(F) All this question wants you to do is find the point at which the four lines in Figure 1
are the closest to one another. That would be the starting point, which represents 8:15–
9:25.

Question: 2
Correct Answer: C
Explanation:
(C) The text explains that lower numbers on the behavioral scale indicate better
behavior, so all this question is asking you to do is find the point (out of the choices)
where the line representing the Adderall kids (the one with the squares)
is not the lowest. (Even if you didn't get that, choice C is still the only one of the choices
where the line representing the Adderall kids is in a different place relative to the other
four lines—when in doubt, pick the odd one out.)

Question: 3
Correct Answer: J
Explanation:
(J) All this question wants you to do is find the point in Figure 2 where the line
representing the double-dose Ritalin kids (the one with the triangles) is highest, which is
over "appetite loss" (as it is for all the lines except the one representing the placebo
kids).

Question: 4
Correct Answer: A
Explanation:
(A) The text explains that lower numbers on the behavioral scale indicate better
behavior, so all this question wants you to do is find the line in Figure 1 that is highest at
the 3:35–4:45 mark, and it is the one representing the placebo kids (the one with the
diamonds).

Question: 5
Correct Answer: F
Explanation:
(F) According to the legend for Figure 1, the group with a 5 behavior rating corresponds
to P—placebo.
Made by: Shahd A.Gaber

Question: 6
Correct Answer: C
Explanation:
(C) In Figure 1, the effectiveness order of the medications from most to least effective is
A1, R2, R1. In Figure 2, the side-effects order of the medications from fewest to most is
R1, R2, A1. The medication choice that "splits the difference" is R2.

(Test 76-2)
Question: 7
Correct Answer: F
Explanation:
(F) The y-axis (vertical line) in Figures 1 and 2 indicate cells per culture dish (millions),
and the last sentence in the paragraph indicates results after three days are shown in
the graphs.

Question: 8
Correct Answer: D
Explanation:
(D) The lines for cell types A and B intersect at 0.4 mg/ml; thus the best answer must
have both cell types included.

Question: 9
Correct Answer: F
Explanation:
(F) An extrapolation of the cell type B line to where it would intersect should 25 uM be
placed on the data graph would yield 4 million cells per culture dish.

Question: 10
Correct Answer: B
Explanation:
(B) The question is asking you to locate the instances listed that would yield cell type A
to be above cell type C and then find the one time (EXCEPT) when cell type C would be
above cell type A. Thus the glucose at 0.2 mg/ml does have cell type A above cell type
C but the 15 uM of calcium has cell type C above cell type A and that is the exception
the question would have us select.
Made by: Shahd A.Gaber

Question: 11
Correct Answer: H
Explanation:
(H) Cell type B in calcium indicates an increase, whereas cell type B in glucose
indicates a plateau or leveling of growth. The question asks for growth. Glucose would
result in growth and then stagnation. Thus, the statement of calcium being more
effective is in line with what the data shows in this particular instance.

(Test 78-2)
Question: 6
Correct Answer: H
Explanation:
(H) The bar representing 500+ size operations is the lowest in Figure 1, which
represents total losses by percentage.

Question: 7
Correct Answer: C
Explanation:
(C) The bar representing CCD losses (the dark one) due to weather is the lowest bar in
Figure 2, which plots the data for suspected causes of losses.

Question: 8
Correct Answer: F
Explanation:
(F) The bar representing CCD losses (the dark one) due to starvation is the highest bar
in Figure 2, which plots the data for suspected causes of losses.

Question: 9
Correct Answer: D
Explanation:
(D) In Table 1 (which plots only CCD losses) the largest loss percentages are due to
starvation in the smaller operations.
Made by: Shahd A.Gaber

Question: 10
Correct Answer: G
Explanation:
(G) All this question wants you to do is identify the suspected causes in Figure 2 that
have CCD bars (the dark ones) higher than non-CCD bars (the light ones). The two that
do are weak in fall and queen death.

Question: 11
Correct Answer: C
Explanation:
(C) For three of the suspected causes in Figure 2—starvation, weather, and pests—the
non-CCD bars are higher than the CCD bars. It would help you here to remember the
answer to question 38. (It is often the case on the ACT Science that remembering
answers to earlier questions on a passage can help you on later ones!)

Question: 12
Correct Answer: J
Explanation:
(J) The scientists surveyed beekeepers to study CCD. The results of both Studies 1 and
2 show numbers related to CCD. Obviously, the beekeepers also had some colonies
that did not experience CCD.

(Test 80-2)
Question: 6
Correct Answer: B
Explanation:
(B) Although no official distinction between a "small" lemon and a "large" lemon is given,
the lemons on the left-hand side of the x-axis (horizontal) in Figure 1 are smaller than
those on the right-hand side, and the highest peak (representing greater sucrose
formation) of any line occurs in the albedo tissue line (dark circles) on the left-hand side.

Question: 7
Correct Answer: G
Explanation:
(G) The line representing sucrose production in the vesicles goes up between the
diameters of 20 and 30 mm, and then down between 30 and 50 mm: in other words, the
sucrose production increases and then decreases.
Made by: Shahd A.Gaber

Question: 8
Correct Answer: C
Explanation:
(C) The question is simply asking you to look at Figure 2 and compare the length of the
bars to determine which of the four answer choices represents the greatest difference in
bar height. Of the four answer choices, the greatest difference is between flavedo tissue
and vesicle tissue in lemons.

Question: 9
Correct Answer: F
Explanation:
(F) In Table 1, the number representing sucrose formed from orange juice (0.12) is
higher than the number representing sucrose formed from lemon juice (0.4).

Question: 10
Correct Answer: C
Explanation:
(C) By comparing the appropriate bars in Figure 2, we see that sucrose production in
albedo tissue of oranges is roughly half that of albedo tissue in lemons. Since Figure 1
indicates that sucrose production in the albedo tissue of a 30-mm lemon is around 17,
we can intuit that sucrose production in an orange of the same size would be roughly
half that, and 9 is the only one of the choices that is anywhere close to half of 17.

Question: 11
Correct Answer: J
Explanation:
(J) The paragraph at the beginning of Study 2 explains that albedo is just the name for
the "white peel layer" of a fruit, and according to Figure 2 it is indeed true that more
sucrose is formed in the albedo tissue of a lemon than in any other part.
Made by: Shahd A.Gaber

(Test 81-2)
Question: 6
Correct Answer: G
Explanation:
(G) The arrows in Figure 1 show how much carbon is transmitted in the global carbon
cycle. There is an arrow at the top of the figure, in the center, which illustrates carbon
being transmitted from the ocean to the atmosphere, and the number next to the arrow
is 90.6.

Question: 7
Correct Answer: C
Explanation:
(C) This is correct because of the data presented in Figure 2. The right side of the figure
is labeled Particles, and the left side of the figure is labeled Dissolved, with the line
dividing them at 0.2 micrometers. Algae is the only type of carbon listed that is on the
right side of the figure, making it the only type of carbon that is NOT a dissolved carbon.

Question: 8
Correct Answer: F
Explanation:
(F) This is correct because of the information presented in Figure 3. The y-axis
represents ocean depth and the x-axis on the top of the figure represents DOC. The key
identifies the line with circular markers as the one for DOC. Trace the line representing
DOC down to determine what happens as the depth increases. It goes straight down for
a while, remaining constant, then it moves to the left, decreasing the amount of DOC.
Then it moves to the right, increasing the amount of DOC.

Question: 9
Correct Answer: D
Explanation:
(D) This is correct because of the information presented in Figure 2. The line at the
bottom of the figure represents the size of the carbon fragments. The third dash from
the left is labeled 0.01. Trace your finger up from that dash to find that two types of
dissolved carbon are listed: small polymers and proteins, making choice D the correct
answer.
Made by: Shahd A.Gaber

Question: 10
Correct Answer: G
Explanation:
(G) This is correct because of the data presented in Figure 3. The y-axis represents
ocean depth. The key shows that the line with the square markers represents CDOM.
The x-axis on the bottom of the figure represents CDOM. Find the dash on the y-axis
labeled 100 and trace your finger over to the line with the square markers. Then trace
your finger down to the x-axis to find that 0.12 is the best answer.

Question: 11
Correct Answer: D
Explanation:
(D) Figure 2 displays which types of carbon fragments are considered dissolved and
which are particles. Organic matter is living, whereas inorganic matter is not. Polymers
are not living, so they are inorganic carbon, and they are displayed on the left side of
Figure 2, under the heading Dissolved.

(Test 82-1)
Question: 1
Correct Answer: H
Explanation:
(H) The second paragraph begins by stating that Researcher 1 expected glucose to
have an effect on alga respiration, and the third paragraph begins by stating that
Researcher 2 expected light to have an effect

Question: 2
Correct Answer: B
Explanation:
(B) For this question it is necessary either to know or to intuit that control is the term for
the version of a scientific experiment in which nothing has been altered, and that is run
so that the experimenters can have something to compare the results of the
experiments in which something has been altered. Other than that, it is a standard "Go
Up, Hit Line, Go Over" question. The solid line in Figure 1 representing the control
(Chlorella to which nothing has been added) is at around 18 mm 3 on the y-axis (vertical)
after 30 minutes.
Made by: Shahd A.Gaber

Question: 3
Correct Answer: F
Explanation:
(F) Researcher 1 predicted in the paragraph that the addition of glucose would inhibit
respiration even further, and Figure 1 shows that the lines representing HCN and H 2S
plus glucose are indeed lower than the lines representing HCN and H2S alone (so
Researcher 1 was right). The question asks what effect a solution with 2 percent
glucose would have, and since the paragraph established that the previous solutions
contained 1 percent glucose, that is twice as much. Twice as much glucose should drop
the lines down to roughly half their current levels. The dotted line representing glucose +
HCN is currently at around 28 mm3 after 20 minutes, and so 15 mm3 would be around
half as much (although it is not exactly half as much, 15 mm 3 is the only one of the
choices that is substantially lower at all).

Question: 4
Correct Answer: D
Explanation:
(D) Neither of the lines representing Chlorella respiration altered direction as a response
to the presence or absence of light, and this did not support Researcher 2's hypothesis.

Question: 5
Correct Answer: G
Explanation:
(G) Researcher 1 hypothesized that the addition of glucose would result in
the Chlorella consuming less oxygen, and so an experiment in which the addition of
glucose causes Chlorella to consume more would weaken his viewpoint.

Question: 6
Correct Answer: B
Explanation:
(B) The substance that had the least effect would be whatever substance is represented
by the line that is closest to the solid control line. After 30 minutes, the line that is
closest to the control line is the dotted line with circles on it, which represents H 2S.

Question: 7
Correct Answer: J
Explanation:
(J) In Figure 3, the solid line representing nitrogen is higher than the dotted line
representing carbon monoxide. Therefore, it is true that Chlorella cells suspended in
Made by: Shahd A.Gaber

nitrogen consume more oxygen than cells suspended in carbon monoxide (in other
words, all of the choices are false except for that one).

(Test 83-1)
Question: 1
Correct Answer: B
Explanation:
(B) When there are two recessive L alleles in the genotype, the leaf texture is wrinkled,
so it can be determined that the L gene affects leaf texture.

Question: 2
Correct Answer: H
Explanation:
(H) The last row in Table 2 lists the phenotypes for this genotype.

Question: 3
Correct Answer: A
Explanation:
(A) The genotype would contain two recessive P alleles. Table 3 contains the data for
the four different crosses performed. It can be determined from Table 2 that if a plant
has 3 pods, the genotype is made up of two recessive P alleles.

Question: 4
Correct Answer: J
Explanation:
(J) It can be determined from Table 2 that the genotype pp results in 3 pods. Table 3
shows that 100% of the offspring for Cross 3 had 3 pods, so 100% of the offspring must
have the genotype pp.

Question: 5
Correct Answer: D
Explanation:
(D) Table 2 displays all the possible genotypes and the associated phenotypes. There
are only two rows where the phenotypes are 3 pods and normal leaves, and the
associated genotypes listed are ppLL and ppLl. All other answers can be eliminated.
Made by: Shahd A.Gaber

(Test 83-2)
Question: 6
Correct Answer: G
Explanation:
(G) Find Table 1. Look at the column labeled Amount of Chlorophyll per Experiment.
The bottom two rows are the data for Experiment 3. The sample exposed to UVR is
labeled 3-UVR. Follow that row until you come to the column with the heading 4, for day
4. The amount of chlorophyll measured was 1.3.

Question: 7
Correct Answer: A
Explanation:
(A) We compare the amounts of chlorophyll measured in Experiment 1 for the sample
exposed to UVR and the sample not exposed to UVR, labeled PAR. For each
measurement, the amount of chlorophyll found in the sample exposed to UVR is either
the same as, or lower than, the PAR sample. The same is true if you compare the data
from Experiments 2 and 3. Therefore, it can be concluded that UVR exposure inhibits
chlorophyll production.

Question: 8
Correct Answer: G
Explanation:
(G) Look at Figure 3. The diatoms exposed to UVR are labeled UVR. The bars on the
graph for UVR are gray. If you compare the two bars for diatoms, you will see that the
UVR bar is much lower than the PAR bar. In fact, the UVR bar for diatoms is the lowest
bar on the graph, making this the correct answer.

Question: 9
Correct Answer: D
Explanation:
(D) The only experiment that continued for eight days was Experiment 3. If you look at
the data from Experiment 3, you will see that the sample not exposed to UVR, labeled
3-PAR, measured more than twice the amount of chlorophyll on day 8 than on day 6.
Assume Experiment 1 would produce similar results. On day 6 the sample 1-PAR
measured 17 for chlorophyll. Day 8 would be more than double that amount, so 36 is
the best answer.
Made by: Shahd A.Gaber

Question: 10
Correct Answer: J
Explanation:
(J) Look at Figure 2. According to the legend, the bars that show the number of dead
cells found in the samples exposed to UVR are white. The white bars are higher for both
diatoms and flagellates.

(Test 84-1)
Question: 1
Correct Answer: C
Explanation:
(C) Look at the section under the heading Researcher 1. The third paragraph lists the
demographic parameters that can be monitored in seabirds. Mortality rates are not
listed.

Question: 2
Correct Answer: J
Explanation:
(J) Look at the section under the heading Researcher 1. The second paragraph makes
this point.

Question: 3
Correct Answer: C
Explanation:
(C) Look at the section under the heading Researcher 2. The second paragraph
addresses this issue and mentions that placing bands on penguins has been shown to
reduce both breeding success and survival rates.

Question: 4
Correct Answer: F
Explanation:
(F) Look at the section under the heading Researcher 1. The last sentence of paragraph
1 supports this answer.
Made by: Shahd A.Gaber

Question: 5
Correct Answer: C
Explanation:
(C) The first paragraph of the passage discusses indicators, and there is a 4-point list of
important criteria for an effective indicator. Researcher 2's views state that seabirds do
not fit the first three criteria and do not address the fourth item in the list. Look at the
section under the heading Researcher 2 to understand how his or her views are related
to each point. Point 1—the last sentence of paragraph 1 explains why seabirds don't
meet this criterion. Point 2—the first sentence of paragraph 2 explains why seabirds
don't meet this criterion. Point 3—the first sentence of paragraph 3 explains why
seabirds don't meet this criterion.

Question: 6
Correct Answer: F
Explanation:
(F) When the amount of prey is low, so is the number of seabirds. Look at the section
under the heading Researcher 1. The last sentence of paragraph 1 makes it clear that
as there is less prey available, the seabird population will decline. This is a direct
relationship and the graph to reflect that will be a diagonal line.

Question: 7
Correct Answer: C
Explanation:
(C) Researcher 2 doesn't mention the cost associated with using seabirds as indicators.
The last paragraph of Researcher 1's point of view states that seabirds are…cost-
effective.

(Test 85-2)
Question: 7
Correct Answer: A
Explanation:
(A) Look at Table 1 and find the largest number, which is 26. Follow the column up to
find that the location is Iquique.

Question: 8
Correct Answer: G
Explanation:
(G) Look at the middle column in Figure 2, which is labeled Coquimbo. Trace your finger
to the left from the top of the bar to find that the average egg size is between 0.15 and
0.2 mm.
Made by: Shahd A.Gaber

Question: 9
Correct Answer: C
Explanation:
(C) Look at Figure 3. Find the tallest bar, which represents the average SMR. The bar
on the left is the tallest and it is a solid black bar. The legend tells you this represents
the TC1 thermal category. Now look down to find the label on the x-axis, which is
Iquique.

Question: 10
Correct Answer: J
Explanation:
(J) Several things must be consulted to answer this question. Table 1 contains the
temperatures that the crabs were exposed to at each location. The text in paragraph 2
defines TC2 as the average maximum temperature, so you know you need to pay
attention to the numbers in that row of the table. In addition, Figure 1 must be consulted
because it contains information about the size of the crabs. Table 1 shows that crabs at
Iquique were exposed to the highest average temperatures at 22, then Coquimbo at 20,
and Concepcion at 17. Figure 1 shows that the average size of the crabs was smallest
at Iquique, slightly larger at Coquimbo, and then largest at Concepcion.

Question: 11
Correct Answer: C
Explanation:
(C) The information in Table 1 and Figure 2 needs to be analyzed to answer this
question. Table 1 shows that the temperatures were highest at Iquique, then Coquimbo,
and lowest at Concepcion. The control temperature was only 1° lower than the average
maximum temperature at Concepcion. Using this information, and looking at Figure 2, it
can be determined that at lower temperatures, the egg volume is greater. So, if
scientists had measured the egg volume of crabs in the control groups, they would have
most likely found larger sizes than at any of the sites measured.

Question: 12
Correct Answer: J
Explanation:
(J) Look at the information in Table 1 and Figure 3 to answer this question. For both
Iquique and Concepcion the black column in Figure 3 is the highest, but for Coquimbo
the white column is the highest. So there is no clear relationship between temperature
exposure and SMR.
Made by: Shahd A.Gaber

(Test 86-2)
Question: 6
Correct Answer: F
Explanation:
FTable 1 lists the distances in between Genes W and X, X and Y, and W and Z. Only
(F) features one of these pairs.

Question: 7
Correct Answer: C
Explanation:
C Look up the values on Figure 2. When the number of proteins is 3,500 molecules per
cell, the frequency of HR is 150 events.

Question: 8
Correct Answer: H
Explanation:
H Look up Scientist 2 on Figure 3. Gene V is closer to Gene X and farther from Gene Z,
so it will be between Genes X and Y. Choice (J) is a trap answer and is wrong because
based on the values in Table 1 and Scientist 2's model, the distance in between Genes
Z and Y is 5 centimorgans.

Question: 9
Correct Answer: D
Explanation:
D Use the distances established in Table 1 to calculate each Scientists' distance
between Genes W and Y. Scientist 1's model would make the distance 50
centimorgans; Scientist 2's model would make the distance 60 centimorgans; Scientist
3's model would make the distance 10 centimorgans; and Scientist 4's model would
make the distance 10 centimorgans. Only Scientist 3 and 4 have values consistent with
each other.

Question: 10
Correct Answer: J
Explanation:
J The introduction lists situations for which recombination can occur outside of sexual
reproduction, including when repair is necessary after exposure to radiation.
Made by: Shahd A.Gaber

(Test 88-1)
Question: 1
Correct Answer: C
Explanation:
C Find the common variable between the figures: time to reach equilibrium is on Figures
2 and 3. On Figure 3, Mixture #3 has peaks at 20 and 30 minutes, as do Solute #3 and
Solute #5 on Figure 2. Choice (A) is a trap answer and is wrong because the time to
reach equilibrium (10 min) matches Mixture #2, not Mixture #3.

Question: 2
Correct Answer: F
Explanation:
F Look up the value on Figure 2. Solute #1 has the smallest molecular mass, therefore
the shortest amount of travel to attain equilibrium, which is supported by the quickest
time in Figure 2.

Question: 3
Correct Answer: C
Explanation:
C Use the values from Table 1 to identify the molecular masses of Solute #3 (2,000)
and Solute #4 (10,000). Eliminate (B) and (D). To identify the molecular mass of Mixture
#2, use the common variable between Experiments 1 and 2: time to reach equilibrium.
Since Mixture #2 takes 10 minutes, it must have a smaller molecular mass than both
Solute #3 and Solute #4. The trends in Table 1 and Figure 1 show that as molecular
mass increases, the time to reach equilibrium also increases.

Question: 4
Correct Answer: G
Explanation:
G Read if and when you can't answer a question from the figures. However, even
without reading, you can use POE to eliminate (F) and (J) because Table 1 disproves
each choice's statement about the molecular mass. The introduction clarifies that
solutes migrate across a membrane in simple diffusion. Use common sense to
connect diffuse easily with the time to reach equilibrium. Figure 2 shows that Solute #3
reached equilibrium more quickly than did Solute #4.
Made by: Shahd A.Gaber

Question: 5
Correct Answer: A
Explanation:
A Use the molecular masses provided in Table 1: Solute #1 has a molecular mass of
160 amu. The common variable between the solutes and mixtures of time to reach
equilibrium shows that Mixture #1 takes less time to reach equilibrium (5 min) than
Solute #1 takes (10 min). Mixture #1 will likely have a smaller molecular mass than 160
amu. Mixtures #2 and #3 take the same time or more time than Solute #1 and will have
the same or greater molecular mass. Choice (D) is wrong because both the text in the
introduction and the data provided in the figures establishes the relationship between
molecular mass and time to reach equilibrium.

Question: 6
Correct Answer: J
Explanation:
J Read the question carefully: Compare the number of molecules in two equal weights.
The molecular mass in amu and weight in grams for Solute #1 is less than the
molecular mass in amu and weight in grams for Solute #5. Therefore, when the weights
are equal, there must be far more molecules of Solute #1 than molecules of Solute #5.
Imagine if you compared a teaspoon of sugar with a teaspoon of Quaker's oatmeal.
There would be many more grains of sugar than grains of oatmeal. If time doesn't allow
the reasoned logic to arrive at the correct answer, at least use POE to eliminate (F) and
(H) because the reasons given are disproven by Table 1.

(Test 88-2)
Question: 7
Correct Answer: A
Explanation:
A Look up the values on Figure 1, which features the variables billions of years before
present day and % composition in atmosphere. As the number of years increases from
4 billion to 3 billion years ago, the percent composition of O2> in Study 2 (solid-line
curve) increases. Choice (C) is a trap answer and is wrong because the percent
composition decreases for H2>O. Use the key to confirm the correct curve.
Made by: Shahd A.Gaber

Question: 8
Correct Answer: H
Explanation:
H Look up the values on Figure 1, and use the key to confirm the correct curve for
Study 1 H2>O vapor (dash-dash-dot). The steepest part of the curve is from 1.5 and 1
billion years ago.

Question: 9
Correct Answer: B
Explanation:
B Look up H2>S and N2> for Study 2 on Table 1. For volcanic eruption models 2–4, the
percent of H2>S is greater than 3. Even though the percent is only 2 for volcanic
eruption model 1, it is likely that H2>S would be 3%. None of the volcanic eruption
models has a percentage greater than 2 for N2>, so it is unlikely that N2> would be 3%.
Choice (B) provides the correct order of Yes for H2>S and No for N2>. Choice (C) is a
trap answer and is wrong because it swaps the order of Yes and No for the two gases.

Question: 10
Correct Answer: H
Explanation:
H Look up the values for Study 2 on Table 1. H2> is 35% in volcanic eruption model 3
and 30% in volcanic eruption model 4. N2> is 2% in both models, and H2>O vapor is 35%
in both models. In a new trial with H2> at 33%, the results for H2>O vapor would be 35%,
the same as in volcanic eruption models 3 and 4.

Question: 11
Correct Answer: D
Explanation:
D The question defines aerobic and anaerobic. Use this information to find a correct
answer that matches the correct term with level of O2>, (D). The level of H2>O is
irrelevant to the level of O2>, which means both (A) and (C) can be eliminated. Choice
(B) is wrong because aerobic organisms need O2> to survive.

Question: 12
Correct Answer: H
Explanation:
H Look up the values on Figure 1. 4 billion years ago, Study 2 (dashed-line curve)
shows that H2>O was 40%. H2>O was 30% 2.5 billion years before the present day, 75%
of its 4-billion year level, a time lapse of 1.5 billion years.
Made by: Shahd A.Gaber

(Test 89-2)
Question: 7
Correct Answer: C
Explanation:
C Use what you know from biology — mitosis refers to the splitting of a cell into two
equal parts. If you weren't sure of this, use the blurb as a hint—it refers to thedividing
cell cycle. This question is asking for the picture of what happens as mitosis nears
completion, or in other words, the picture that shows the closest thing to two separate
entities, which the picture for Cell 3 does.

Question: 8
Correct Answer: J
Explanation:
J The graph shows a direct relationship between toxin concentration and AI, the
apoptotic index. Therefore, if the AI is approximately 0.007 (or 0.7%) at 80 ppm, it must
be greater than that value at 90 ppm, making (J) the only possible answer.

Question: 9
Correct Answer: D
Explanation:
D As the actively dividing cell cycle progresses, the cell gets larger before its two halves
split into equal parts. The first stage must therefore show the smallest and least divided
picture, as (D), Cell 4, does.

Question: 10
Correct Answer: G
Explanation:
G According to Figure 2, there were 225 cells in cycle stage G2>, and there were 75
cells in stage S. There were therefore 3 times as many cells in cycle stage G 2> as there
were in stage S, as (G) indicates.

Question: 11
Correct Answer: A
Explanation:
A The text above the graph indicates that the graph charts one thousand actively
dividing fibroblast cells. There were just over 525 cells in cycle stage G1. Therefore, the
proportion of all the cells that were in cycle stage G1 was approximately 525 of 1,000, or
(A).
Made by: Shahd A.Gaber

(Test 90-1)
Question: 1
Correct Answer: F
Explanation:
F If you're unsure whether to answer Yes or No, use the reasons. Choices (G) and (J)
can be eliminated because Patient 4 and Patient P do have the same blood-smear
findings according to Table 1. Judging from Table 1 alone, we have all the necessary
evidence to say that Patient 4 and Patient P are the same, thus making (F) the correct
answer.

Question: 2
Correct Answer: C
Explanation:
C This question requires a bit of outside knowledge, unless you use POE aggressively!
First, choice (A) suggests that hemoglobin breaks down at a pH of 8.6, but this
experiment is measuring hemoglobin, so the scientists would not want it broken down.
Always give special consideration to opposites—in this case, (C) and (D). Then, the
scientists likely used this pH level because the process they were observing
occurs best at this level, making (C) the correct answer.

Question: 3
Correct Answer: H
Explanation:
H Use POE. Table 1 indicates that Patient 4's blood contains sickle cells, which means
that this patient must have at least one of the alleles that causes sickle cell anemia. As
the question indicates, the alleles that cause sickle cell anemia are HbS and HbC. Pairs
of alleles that contain only HbA will not have sickle cell anemia, thus eliminating (I),
which contains only HbA, and therefore eliminating (G) and (J). The remaining two
options, (II) and (III), both contain either HbS and HbC, so both are possibilities, making
(H) the correct answer.

Question: 4
Correct Answer: B
Explanation:
B Match the peaks in the various lines. Patient P's single peak most closely resembles
that of Patient 2, thus making (B) the only possible answer.
Made by: Shahd A.Gaber

Question: 5
Correct Answer: J
Explanation:
J The figure shows a positive electrode on the left side and a negative electrode on the
right. Similar charge characteristics must therefore be similar positions relative to given
electrodes. Of the choices listed, peaks X and Z seem to be a similar distance from the
positive electrode, making (J) the best answer. Peaks W and Y are also similarly
positioned, but no answer choice indicates this pair.

Question: 6
Correct Answer: A
Explanation:
A Use POE. Opposite charges attract each other, eliminating (B) and (D). In addition,
"the left" refers to the side of the positive electrode. Because opposite charges attract
each other, the peaks closest to the positive electrode must be negative, as (A)
indicates.

(Test 92-2)
Question: 8
Correct Answer: D
Explanation:
D Use POE. If albumin molecules do not pass through the selectively permeable
membrane, the albumin solution on the left side will not pass through to the right,
meaning the solution levels on the left cannot fall, eliminating (A) and (C). Then,
because there is no glucose-cupric combination, there is no reason for there to be a red
solution, eliminating (B). Only (D) remains and is the correct answer.

Question: 9
Correct Answer: F
Explanation:
F In Experiment 1, the cupric solution is able to pass through the membrane to produce
a red color with the glucose solution. In Experiment 2, the cupric solution is able to pass
through the membrane to form a blue-colored solution with the pure water. It can be
inferred, then, that cupric was able to pass through the membrane in both experiments
and into both glucose and pure water, as (F) suggests.
Made by: Shahd A.Gaber

Question: 10
Correct Answer: C
Explanation:
C Read the first lines of each experiment. Experiment 2 states that Cupric ion solution is
poured in the left, and Experiment 3 states that Glucose solution is poured in the left.
These statements match with the information in (C).

Question: 11
Correct Answer: G
Explanation:
G Because mixing glucose and cupric ions results in a red solution, a membrane that
allows all solutions to pass through would result in the mixing of all solutions. In
Experiment 1, glucose is poured in the left and cupric in the right. With a membrane that
allows both these solutions to pass through, the solutions will mix throughout the tube,
creating a red solution in all parts of the tube.

Question: 12
Correct Answer: A
Explanation:
A Because the solution on the left becomes red, cupric ions must be able to pass
through the membrane, and because the water levels change, water must be passing
through the membrane as well. If glucose were able to pass through the membrane,
both sides of the tube would be filled with red-colored solution. Therefore, it can be
inferred that glucose is larger than both cupric and pure water, but it is not possible to
determine the relative sizes of cupric and pure water, as (A) suggests.

Question: 13
Correct Answer: G
Explanation:
G Experiment 1 states, cupric ion solution is poured on the right. The information above
Experiment 1 states, Water and glucose solutions are colorless while cupric ion
solutions are blue. Therefore, before the experiment begins, the right side, containing
only cupric ion solution, must have been blue, as (G) suggests.
Made by: Shahd A.Gaber

(Test 93)
Question: 1
Correct Answer: C
Explanation:
The best answer is C. According to Figure 2, Amino Acid A (the solid line) has the
highest rate (on the y-axis) at a water volume between 10 mL and 15 mL. This volume
is closest to answer choice C, 12 mL.

Question: 2
Correct Answer: H
Explanation:
The best answer is H. Amino Acid A and Amino Acid B have the same reaction rate
when the two lines cross on the graph. This happens at a water volume of
approximately 50 mL.

Question: 3
Correct Answer: A
Explanation:
The best answer is A. Figure 3 shows that the reaction rate of Amino Acid B is not
dependent on the residue concentration. Figure 4 shows that the reaction rate of Amino
Acid B is dependent on the amino acid concentration. Therefore, the figures do not
support the researcher's claims that the reaction of Amino Acid B is dependent on both
concentrations.

Question: 4
Correct Answer: G
Explanation:
The best answer is G. According to Figure 4, Amino Acid A always has a lower rate of
reaction than Amino Acid B. This does not support the researcher's claims that the
reaction rate for Amino Acid A at any given concentration will always be greater than
Amino Acid B for the same concentration. Answer choices F and H can be eliminated
because the effect of residue concentrations on the rate of reaction is not shown in
Figure 4.
Made by: Shahd A.Gaber

Question: 5
Correct Answer: A
Explanation:
The best answer is A. According to the figure, the water volume of peak reaction rates
for amino acids in the muscles (glycine) is higher (35 mL) than that of amino acids in
DNA (alanine–20 mL).

(Test 97)
Question: 1
Correct Answer: F
Explanation:
The best answer is F. In Experiment 1, the control group (the group that the results are
compared to) included the rosebushes that were grown in soil with no pesticide added.
Question: 2
Correct Answer: D
Explanation:
The best answer is D. Answer choice D is the correct answer because if rosebushes
generally die when pesticides are directly applied to them, it would make sense to apply
the pesticides to the soil.

Question: 3
Correct Answer: G
Explanation:
The best answer is G. If there is a direct correlation between plant weight and the
number of petals on the flowers, the lower the average weight of the rosebush, the
lower the number of petals. A rosebush grown in Soil Type 2 would have the lowest
number of petals with Pesticide A at 35 ppm (23.6 oz).

Question: 4
Correct Answer: D
Explanation:
The best answer is D. If Pesticide C was used to treat a rosebush, no prediction could
be made on the basis of the previous results because no information about Pesticide C
was given.
Made by: Shahd A.Gaber

Question: 5
Correct Answer: G
Explanation:
The best answer is G. The results of Experiment 2 are shown in Table 2. According to
the table, the average weight of the rosebush was lowest in every row for Soil Type 1
and Pesticide A.

(Test 98)
Question: 1
Correct Answer: D
Explanation:
The best answer is D. According to Table 1, the mass of the precipitate in Test Tube 1
was 4.3 mg. The difference between this test tube and the other test tubes is that it had
a shorter incubation time (0 min) than other test tubes at the same temperature water
bath.
Question: 2
Correct Answer: F
Explanation:
The best answer is F. In Experiment 1, "7 milliliters of a peptide (a neurotransmitter)
solution was added." In Experiment 2 "8 mL of peptide solution was added" to the test
tubes. One difference between the two experiments is the amount of peptide solution
used. A larger volume of the peptide solution was used in Experiment 2, answer choice
F.

Question: 3
Correct Answer: C
Explanation:
The best answer is C. According to Table 2, as the pH level increases (going down the
table), the mass of the precipitate first increases from 2.5 mg to 6.2 mg and then starts
to decrease. This information best supports answer choice C.

Question: 4
Correct Answer: F
Explanation:
The best answer is F. According to Table 1, the masses of the precipitate for Test Tube 1
(incubation period 0 min) and Test Tube 2 (incubation period 5 min) were 4.3 mg and 3.9 mg,
respectively. If Tube 5 is incubated for 5 min and has a mass precipitate of 3.6 mg, the mass is
just below that of Tube 2 for the same incubation period. It could be predicted that if Tube 5
were not incubated, the mass of its precipitate would equal 4.1 mg, just below that of Tube 1's
mass.
Made by: Shahd A.Gaber

Question: 5
Correct Answer: A
Explanation:
The best answer is A. According to Table 1, 3.9 mg of precipitate was collected from
Test tube 2 at a temperature of 25°C after a 5-minute incubation period. This is the
greatest amount of precipitate yielded among the combinations shown in the answer
choices.

Question: 6
Correct Answer: H
Explanation:
The best answer is H. To answer this question, you must look at the results in Tables 1
and 2. According to Table 1, the mass of the precipitate is the least for test tubes with
an incubation period of 15 min for 30°C (1.4 mg). According to Table 2, the mass of the
precipitate is least for the test tube with a pH of 2. Therefore, if combined, these two
conditions would likely produce the lease amount of precipitate.

(Test 99)
Question: 1
Correct Answer: D
Explanation:
The best answer is D. According to Table 1, Salmonella bacteria had the greatest
percent growth at a pH level of 5. This information suggests a pH level near 5 might
best contribute to the growth of the bacteria.

Question: 2
Correct Answer: H
Explanation:
The best answer is H. To answer this question you must look at the results shown in all
of the tables and choose the conditions that create the greatest percent growth of
bacteria. In Table 1, a pH level of 5 created the highest percent growth (81%) of the
bacteria. In Table 2, the organic compound in Dish 1 created the greatest percent
growth (37%) of the bacteria. In Table 3, a temperature of 40°C created the greatest
percent growth (83%) bacteria. By combining these three conditions you would expect
to get the greatest percent growth.
Made by: Shahd A.Gaber

Question: 3
Correct Answer: A
Explanation:
The best answer is A. Table 1 shows that the Salmonella bacteria reproduced most
efficiently in a pH of 5 (acidic). A conclusion stating that Salmonella bacteria reproduce
most efficiently in an acidic environment would reaffirm the results of Experiment 1.

Question: 4
Correct Answer: G
Explanation:
The best answer is G. To maximize the length of time that bacteria will reproduce, the
bacteria should be provided an unlimited supply of nutrients. In order to make sure the
nutrients are not depleted, the bacteria groups must be regularly resupplied with
unlimited nutrients, answer choice G.

Question: 5
Correct Answer: B
Explanation:
The best answer is B. In a scientific experiment, the independent variable is the factor
that is changed by the experimenter during the experiment. In Experiment 3, the
temperature was changed; each dish had a different temperature. The factor being
studied or observed, in this case the percent of the growth, is the dependent variable.
Whether it changes depends on the manipulation of one or more independent variables.

Question: 6
Correct Answer: J
Explanation:
The best answer is J. Answer choice J is the best answer because it is the only answer
choice that tests the bacteria at intervals. Because bacteria reproduce at a rate that
varies from one stage to the next, by testing at 15-minute intervals you could study the
different stages of growth most effectively.
Made by: Shahd A.Gaber

(Test 105)
Question: 1
Correct Answer: H
Explanation:
The best answer is H. According to the information in Table 1, Grackle E (the heaviest
bird at 350 g) was ranked 3rd—right in the middle—in aggressiveness. In addition,
Table 3 indicates that Grackle A, the lightest bird, had more wins and fewer losses than
did Grackle E, which would suggest that Grackle A is more aggressive than Grackle E.
The data best supports the idea that body weight has no effect on aggression, answer
choice H.

Question: 2
Correct Answer: A
Explanation:
The best answer is A. Table 3 shows each of the grackles' win–loss records. The
grackle with the highest win–loss record was Grackle B (95–10). This information would
suggest Grackle B would have the highest likelihood of mating, answer choice A.

Question: 3
Correct Answer: J
Explanation:
The correct answer is J. To solve this problem, compare the values given with the
values listed in Table 1. Based on those values, the sixth grackle is most similar to
Grackle D, which had an aggression rank of 5. However, because the sixth grackle had
fewer calls, it would be less aggressive, and so would likely have a lower rank.

Question: 4
Correct Answer: D
Explanation:
The best answer is D. According to Experiments 1 and 2, in general, the more calls a
grackle makes, the higher the aggression ranking and the more times the grackle wins.
This information suggests the number of calls is related to the dominance of one male
grackle over other males.
Made by: Shahd A.Gaber

Question: 5
Correct Answer: H
Explanation:
The best answer is H. Table 2 shows the number of times each grackle won over
another grackle. According to the table, Grackle A won over Grackle C a total of 5
times. Grackle C won over Grackle A 20 times. This information suggests the two birds
encountered each other 20 + 5 = 25 times.

Question: 6
Correct Answer: B
Explanation:
The best answer is B. In order to refute the criticism that the order the grackles were
placed in the cage may have affected the aggressiveness of each bird, the experiment
should be repeated several times with different orders each time. Answer choice A
seems plausible, but even though the order is randomized, it might still affect
aggressiveness. By repeating the experiment several times, the criticism will be
overcome.

(Test 106)
Question: 1
Correct Answer: G
Explanation:
The best answer is G. To answer this question, you must look at Table 1 and determine
the amount of growth for each week.

In all three plant types, the greatest growth occurred between Weeks 2 and 4.
Made by: Shahd A.Gaber

Question: 2
Correct Answer: C
Explanation:
The best answer is C. In Experiment 1, Plant type 2 was not studied. According to Table
3, Plant type 2 is from the Northern Forest.

Question: 3
Correct Answer: H
Explanation:
The best answer is H. To answer this question, find the difference in growth between
Week 1 and Week 6 for each plant type:
Plant type 1: 3.7 - 1.2 = 2.5
Plant type 3: 2.0 - 0.6 = 1.4
Plant type 4: 3.5 - 0.9 = 2.6

Question: 4
Correct Answer: B
Explanation:
The best answer is B. To answer this question, you must look at the different in growth
between the plants in Tables 1 and 2.
Plant type 1: 5.1 - 3.7 = 1.4 in.
Plant type 3: 5.4 - 2.0 = 3.4 in.
Plant type 4: 4.9 - 3.5 = 1.4 in.
Fungi had the greatest affect on Plant type 3.

Question: 5
Correct Answer: H
Explanation:
The best answer is H. According to Tables 1 and 2, Plants 1, 3, and 4 all grew more
when the fungi was present in the soil. This information suggests that the presence of
certain fungi in the soil increases plant growth.
Made by: Shahd A.Gaber

(Test 108)
Question: 1
Correct Answer: H
Explanation:
The correct answer is H. You are given that "the order Lepidoptera includes butterflies
and moths. Table 1 is a key for identifying some Lepidoptera in North America."
Because butterflies and moths are not birds, reptiles, or mammals, they must be
insects. Even though you might never have seen the word Lepidoptera before, you can
still correctly answer the question because the term is defined for you.

Question: 2
Correct Answer: B
Explanation:
The correct answer is B. To answer this question, find Agraulis vanillae on Table 1.
According to Table 1, the upper side of Agraulis vanillae wings is orange with black
markings. This species does not have a fuzzy body—Table 1 indicates that you should
skip to Step 3 if the insect has a fuzzy body, bypassing Agraulis vanillae completely.
Likewise, the features mentioned in answer choices C and D are located after Agraulis
vanillae on the table.

Question: 3
Correct Answer: H
Explanation:
The correct answer is H. To answer this question, find the listed traits of
Lepidoptera Y in Table 2, and apply them to the steps in Table 1. Table 2 indicates that
Lepidoptera Y has a slim body; therefore, go to Step 2 and eliminate answer choice F.
Table 2 indicates that Lepidoptera Y has yellow wings with markings on the upper side,
so go to Step 4 and eliminate answer choice G. Table 2 indicates that
Lepidoptera Y has silver markings, so go to Step 6. Because the silver markings
of Speyeyria coronis are round and elongated and Lepidoptera Y has triangular shaped
silver markings, you can deduce that Lepidoptera Y is not a Speyeyria coronis.
Made by: Shahd A.Gaber

Question: 4
Correct Answer: C
Explanation:
The correct answer is C. To answer this question, evaluate each of the answer choices
and eliminate those that are incorrect:
Answer choice A: Step 1—According to Table 2, both W and Z have a fuzzy body;
eliminate answer choice F.
Answer choice B: Step 3—According to Table 2, both W and Z have wings with yellow
on the upper side; eliminate answer choice G.
Answer choice C: Step 7—According to Table 2, W has prominent brown bands on its
underside, and Z has no prominent markings on its underside. Therefore, Step 7 is
where they first diverge.

Question: 5
Correct Answer: F
Explanation:
The correct answer is F. To answer this question, first notice that Automeris
io and Antheraea polyphemus are both listed in Step 8. According to Table 1, to get to
Step 8 the Lepidoptera must have pronounced spots on their hindwings. Note that the
Lepidoptera in the question have different wingspans, so answer choice H is incorrect.
Also, by moving directly to Step 8, Steps 6 and 7 are skipped, so eliminate answer
choice G. The only way to get to Step 5 is to have a fuzzy body, so eliminate answer
choice J.

(Test 110)
Question: 1
Correct Answer: H
Explanation:
The correct answer is H. Scientist 1 supports the use of placebos in clinical research.
According to Scientist 1, placebos "help address the issue of mind over matter, which is
an important issue when working towards treatment for a particular illness." In addition,
Scientist 1 states that, "… the possibility of receiving treatment often provides patients
with a psychological boost." These statements best support answer choice H.
Made by: Shahd A.Gaber

Question: 2
Correct Answer: B
Explanation:
The correct answer is B. Scientist 1 states that, "… the possibility of receiving treatment
often provides patients with a psychological boost. The use of placebos addresses the
question of whether a person's positive attitude may be important in recovery from
illness." These statements best support answer choice B.

Question: 3
Correct Answer: G
Explanation:
The correct answer is G. Both Scientists address the use of placebos in clinical
research; Scientist 1 supports the use of placebos while Scientist 2 argues against the
use of placebos. Both Scientists discuss the placebo effect as a possible result of using
placebos in clinical research. Therefore, the only statement that both Scientists would
agree with is answer choice G

Question: 4
Correct Answer: C
Explanation:
The correct answer is C. Scientist 2 states that, "Due to its capability to skew research
results, the placebo shouldn't be used in clinical research."

Question: 5
Correct Answer: J
Explanation:
The correct answer is J. Scientist 2 believes that the use of placebos encourages
deception in the doctor-patient relationship, which suggests that the use of placebos
would lead to less positive communication between the patients and their doctors.
Because the finding indicates that the opposite is true—in fact, that the patients were
very trusting of their doctors and freely communicated with them—Scientist 2's
viewpoint would be weakened.

Question: 6
Correct Answer: A
Explanation:
The correct answer is A. Scientist 1 states that placebos "establish a control group for
the test treatment in question …" If an observation were made suggesting that patients
in a control group— the group receiving the placebo—did not recover as quickly as
Made by: Shahd A.Gaber

patients who received the medical treatment, Scientist 2's viewpoint against the use of
placebos would be weakened. According to Scientist 2, "placebo-related changes could
be overestimated. Different illnesses, by definition, will react differently to the placebo."
The observation in answer choice A would support using placebos to help determine the
efficacy of a given medical treatment.

Question: 7
Correct Answer: J
Explanation:
The correct answer is J. Because Scientist 1 believes that using placebos is beneficial
in clinical research, Scientist 1 must assume that the use of placebos is safe for most
patients. None of the other answer choices are supported by the passage.

(Test 115)
Question: 1
Correct Answer: F
Explanation:
The correct answer is F. According to the passage, "Contact fungicides, because they
do not penetrate the plant tissue, are more easily washed off the leaf by rain. This
results in a shorter residual control period and more frequent re-application of the
fungicide." Table 1 indicates that boscalid is a contact fungicide, so it should be applied
more frequently than the others listed.

Question: 2
Correct Answer: C
Explanation:
The correct answer is C. The passage states that, "Fungicide applications should not be
initiated after the R4 growth stage." There is nothing in the passage to indicate
that chlorthalonil should not be applied during any growth stage.

Question: 3
Correct Answer: G
Explanation:
The correct answer is G. According to the passage, "research has shown that the most
critical time to protect soybean plants with fungicides is from the R1 through R3 growth
stages. Fungicide applications should not be initiated after the R4 growth stage." This
information best supports answer choice G.
Made by: Shahd A.Gaber

Question: 4
Correct Answer: D
Explanation:
The correct answer is D. According to the passage, "ASR infections generally begin in
the lower leaf canopy … Coverage as dense as 400 spray droplets per square inch is
considered ideal." Therefore, you can predict that if fewer than 400 droplets of any
fungicide were applied, the chances of the plant becoming infected with ASR would
increase.

Question: 5
Correct Answer: H
Explanation:
The correct answer is H. According to Table 2, systemic fungicides are most effective
because they are absorbed into the leaf tissue and translocated throughout the plant.
Table 1 indicates that myclobutanil is a systemic fungicide. Because the question asks
you to put the fungicides in order of increasing effectiveness, myclobutanil should be
last on the list. Eliminate answer choices F and J. The data indicate that contact
fungicides are the least effective and that boscalid is a contact fungicide.
Therefore, boscalid should be first on the list.

Question: 6
Correct Answer: B
Explanation:
The correct answer is B. According to the passage, "research has shown that the most
critical time to protect soybean plants with fungicides is from the R1 through R3 growth
stages. Fungicide applications should not be initiated after the R4 growth stage."
Because growth stage R4 is characterized by a full seed, fungicide should NOT be
applied during the full seed stage.
Made by: Shahd A.Gaber

(Test 116)
Question: 1
Correct Answer: H
Explanation:
The correct answer is H. According to the passage, "sea cucumbers are wormlike and
tend to burrow." The echinoderm pictured most resembles a worm so it is likely a sea
cucumber.

Question: 2
Correct Answer: C
Explanation:
The correct answer is C. According to Table 1, sea lilies are crinoids. Table 2 shows
that sea lilies are most likely found in offshore mud and ooze, which best supports
answer choice C.

Question: 3
Correct Answer: F
Explanation:
The correct answer is F. According to Table 1, the feeding method of halothurian (sea
cucumbers) is to swallow mud and sand; therefore, it is most likely that sea cucumbers
burrow into the mud and sand in search of food. The other answer choices are not
supported by the passage.

Question: 4
Correct Answer: B
Explanation:
The correct answer is B. According to Table 1, asteroids (starfish) are predatory and
use their tube feet to move. Table 2 indicates that starfish can be found on the deep-sea
floor. This information best supports answer choice B.

Question: 5
Correct Answer: G
Explanation:
The correct answer is G. According to the passage, "sea cucumbers are wormlike and
tend to burrow." Table 1 indicates that sea cucumbers feed by swallowing mud and
sand, digesting any organic material, and ejecting whatever is left over. Sea cucumbers
in large numbers could, simply by feeding, alter the physical and chemical composition
of the sea floor. The other answer choices are not supported by the data.
Made by: Shahd A.Gaber

(Test 117)
Question: 1
Correct Answer: C
Explanation:
The correct answer is C. The passage states that the average starting weight of the
guinea pigs in each group was 50 g. According to Table 2, the guinea pigs in Group 5
had the highest average weight after 8 weeks (98 g). Guinea pigs in this group were
given Feed V.

Question: 2
Correct Answer: J
Explanation:
The correct answer is J. According to Table 1, the guinea pigs that were fed a grain-
based feed with vitamin supplements (Group 2, Feed Q) gained an average of 32 grams
over 8 weeks (82 - 50 = 32). Therefore, on average, these guinea pigs gained 4 grams
per week (32 ÷ 8 = 4).

Question: 3
Correct Answer: C
Explanation:
The correct answer is C. According to the question, the guinea pigs in Group 9 would
receive Feed V (the same as Group 5) and a vitamin supplement. According to Table 2
and the information in the passage, the guinea pigs in Group 5 had the greatest
average weight and height after 8 weeks. Additionally, the guinea pigs given a vitamin
supplement (like in Group 6) showed more growth than the guinea pigs that did not
receive vitamin supplements (such as Groups 7 and 8). Therefore, if Group 9 is given
Feed V with vitamin supplements, the guinea pigs will most likely be even larger than
those in Group 5, which best supports answer choice C.

Question: 4
Correct Answer: J
Explanation:
The correct answer is J. The best way to answer this question is to examine each of the
answer choices, and eliminate those that are not supported by the data in Table 2.
Answer choice F: Group 7 is the control group. Group 6 was given Feed W. It is not
true that the guinea pigs in Group 6 are twice as long as the guinea pigs in Group 7 so
eliminate answer choice F.
Made by: Shahd A.Gaber

Answer choice G: Group 5 was given Feed V. It is not true that the guinea pigs in
Group 5 weigh three times more than the guinea pigs in Group 7, so eliminate answer
choice G.
Answer choice H: Group 8 was given Feed Y; guinea pigs in this group did not have
the greatest average length, so eliminate answer choice H.
Answer choice J is correct, because it is true that the guinea pigs in Group 7, who
received Feed X, are similar in both weight and length to the guinea pigs in Group 8,
who received Feed Y.

Question: 5
Correct Answer: B
Explanation:
The correct answer is B. The only difference between Group 3 (the control group) and
Group 4 is that the guinea pigs in Group 4 were fed fruits and vegetables in addition to a
grain-based feed. The students can hypothesize that those fruits and vegetables must
have a relatively low nutritional value, since the guinea pigs in Group 4 did not grow
much more than did the guinea pigs in the control group.

Question: 6
Correct Answer: F
Explanation:
The correct answer is F. According to the results of the experiments, the guinea pigs in
Group 8 gained the least amount of weight (74 - 50 = 24 grams) and had the smallest
increase in average length (23.25 - 20 = 3.25 cm).

(Test 120)

Question: 1
Correct Answer: G
Explanation:
The correct answer is G. According to the passage, Plot B received applications of high-
nitrogen fertilizer, whereas Plot A received applications of low-nitrogen fertilizer. Both
Table 1 and Figure 1 indicate that the leachate from Plot B always contained higher
average nitrogen levels than the leachate from Plot A. This information best supports
answer choice G.
Made by: Shahd A.Gaber

Question: 2
Correct Answer: A
Explanation:
The correct answer is A. The passage states that, "The leaching of high concentrations
of nitrogen into natural waterways can throw off the environmental equilibrium of the
aquatic ecosystem, often resulting in an increase in plant growth that can have a
negative impact on the native fish populations." Therefore, if low-nitrogen fertilizer is
applied, any leachate into surrounding waterways should have low concentrations of
nitrogen, and the native fish populations will not be affected.

Question: 3
Correct Answer: H
Explanation:
The correct answer is H. According to Table 1, the average nitrogen concentration in
the leachate from both plots during 2003 was higher than in previous or subsequent
years. Therefore, more nitrogen was leached from the soil in 2003, suggesting that this
year experienced heavy rainfall during the months of the study.

Question: 4
Correct Answer: B
Explanation:
The correct answer is B. According to Table 1, only Plot A had average nitrogen
concentrations less than 10 mg/L. Because this plot received a fertilizer application
containing 98 kilograms of nitrogen per acre, you can conclude that the Environmental
Protection Agency would consider this application safe.

Question: 5
Correct Answer: H
Explanation:
The correct answer is H. According to the passage, the nitrogen levels in the fertilizer
applied to Plot B were reduced in 2004. However, Table 1 shows that the nitrogen
concentration in the leachate from Plot B was 11.8 mg/L, which is still considered
unsafe. In 2005, the concentration had gone down to a safe 8.2 mg/L. Therefore, you
can conclude that it will take more than one year to reach safe nitrogen levels in the
leachate. The other answer choices are not supported by the data.
Made by: Shahd A.Gaber

(Test 123)
Question: 1
Correct Answer: J
Explanation:
The best answer is J. Table 1 lists characteristics of certain predators. Since "herbivore"
is listed in the table, a herbivore is a predator. The passage also indicates that some
scientists contend that herbivores are predators. According to Table 1, carnivores eat
herbivores, which means that a herbivore is also a prey animal.

Question: 2
Correct Answer: D
Explanation:
The best answer is D. The passage states that the number of prey consumed depends
on the number of prey present as well as the number of predators present. The other
answer choices are not supported by the passage.

Question: 3
Correct Answer: F
Explanation:
The best answer is F. Figure 1 shows that, when the predator population goes up, the
prey population goes down. This indicates an inverse relationship.

Question: 4
Correct Answer: C
Explanation:
The best answer is C. According to Table 1, a deer is herbivorous, so eliminate answer
choice B. Table 1 also indicates that herbivores can be selective eaters, which means
that they may choose to eat a certain type of plant over other types of plants. This best
supports answer choice C.

Question: 5
Correct Answer: G
Explanation:
The best answer is G. To answer this question, find the location on Figure 1 where the
line representing prey reaches its highest point. This occurs during the second year.
Made by: Shahd A.Gaber

(Test 125)
Question: 1
Correct Answer: B
Explanation:
The best answer is B. Figure 1 shows some of the internal anatomy of a sea anemone.
According to this figure, the sea anemone's mouth is located at the center of its body.
The other answer choices are not supported by Figure 1.
Question: 2
Correct Answer: F
Explanation:
The best answer is F. The passage states that zooxanthellae are a food source for the
sea anemone. The passage also indicates that the clown fish helps the sea anemone
by actually cleaning the tentacles. This best supports answer choice F.

Question: 3
Correct Answer: B
Explanation:
The best answer is B. The passage starts out by saying that sea anemones look like
plants. Also, Figure 1 shows that a sea anemone looks more like a plant or a flower
than an animal. Nothing in the passage suggests that a sea anemone looks like a clown
fish, marine algae, or a rock.

Question: 4
Correct Answer: H
Explanation:
The best answer is H. According to the passage, "when ocean currents are strong, the
sea anemone will reduce its internal volume in order to decrease the surface area that
is exposed to the current." This best supports answer choice H.

Question: 5
Correct Answer: C
Explanation:
The best answer is C. Figure 1 clearly shows that the sea anemone's mouth is
connected to its gastrovascular cavity by way of the pharynx. The other answer choices
are different internal parts of the sea anemone, but they do not correctly answer the
question.
Made by: Shahd A.Gaber

(Test 126)
Question: 1
Correct Answer: G
Explanation:
The best answer is G. According to Table 3, the average plant height of the plants
grown in potting soil was 5.3 centimeters, and the average number of leaves on the
plants grown in potting soil was 6. This best supports answer choice G.

Question: 2
Correct Answer: D
Explanation:
The best answer is D. According to Table 2, after 14 days the soil/compost mixture that
yielded the greatest average plant height was mixture 1, with an average plant height of
4.2 centimeters.

Question: 3
Correct Answer: G
Explanation:
The best answer is G. The passage states that, in Experiment 2, "each pot contained a
different soil type, and no compost was used." The other answer choices are not
supported by the passage.

Question: 4
Correct Answer: D
Explanation:
The best answer is D. Based on the data in Tables 2 and 3, soil from near the school
was used in Pot 4 in Experiment 1 and in Pot 3 in Experiment 2. This would most likely
account for the nearly identical results recorded. The other answer choices are not
supported by the data.

Question: 5
Correct Answer: F
Explanation:
The best answer is F. The passage indicates that 4 radish seeds were placed in each
pot in Experiment 1. The only differences between Experiment 1 and Experiment 2 were
the soil types used and the fact that no compost was used. Therefore, the same number
of seeds (4) was planted in each pot in Experiment 2.

You might also like